Suites et séries de fonctions, séries entières

Exercices du dossier Suites et séries de fonctions, séries entières

Exercice 1329

9 novembre 2022 12:19 — Par Patrice Lassère

Bibliographie



[ID: 2531] [Date de publication: 9 novembre 2022 12:19] [Catégorie(s): Géométrie Topologie Continuité Dérivabilité Intégration Suites et séries Suites et séries de fonctions, séries entières Fonctions holomorphes Calcul différentiel Equations différentielles Analyse fonctionnelle Combinatoires et probabilités En cours... ] [ Nombre commentaires: 0] [nombre d'éditeurs: 1 ] [Editeur(s): Emmanuel Vieillard-Baron ] [nombre d'auteurs: 1 ] [Auteur(s): Patrice Lassère ]
Accordéon
Titre
Solution
Texte

Étude d’une série de fonctions *

9 novembre 2022 22:30 — Par Patrice Lassère

[rms]-2006.

On considère une suite de fonctions \((f_n)_n\subset\mathscr C([0,1],\mathbb R)\) vérifiant \[\vert f_n(x)f_m(x)\vert\leq 2^{-\vert n-m\vert},\quad\forall\,n,m\in\mathbb N,\ x\in[0,1].{(\text{$\star$})}\]

  1. Montrer que la série de fonctions \(\sum_n f_n\) converge simplement sur \([0,1]\).

  2. Montrer que la somme \(f=\sum_n f_n\) est bornée sur \([0,1]\).

  3. La convergence est-elle uniforme sur \([0,1]\) ?



[ID: 2887] [Date de publication: 9 novembre 2022 22:30] [Catégorie(s): Suites et séries de fonctions, séries entières ] [ Nombre commentaires: 0] [nombre d'éditeurs: 1 ] [Editeur(s): Emmanuel Vieillard-Baron ] [nombre d'auteurs: 1 ] [Auteur(s): Patrice Lassère ]
Accordéon
Titre
Solution
Texte

Étude d’une série de fonctions
Par Patrice Lassère le 9 novembre 2022 22:30

Soit \(x\in [0,1]\).

Supposons qu’il existe \(n_0\in\mathbb N\) tel que \(f_{n_0}(x)\neq 0\) , avec (\(\star\)) on a \[\vert f_n(x)f_{n_0}(x)\vert\leq 2^{-\vert n-{n_0}\vert},\quad\forall\,n\in\mathbb N,\] soit \[\vert f_n(x)\vert\leq 2^{-n}\dfrac{2^m}{\vert f_{n_0}(x)\vert},\quad\forall\,n\in\mathbb N\] d’où la convergence absolue de la série \(\sum_n f_n(x)\) par critère de comparaison.

Sinon \(f_n(x)=0\) pour tout \(n\in\mathbb N\) et la convergence au point \(x\) est évidente.

Avec la question précédente la convergence simple est absolue et la suite \((\vert f_n\vert)_n\) vérifie aussi (\(\star\)) : comme \(\vert\sum_n f_n\vert\leq \sum_n\vert f_n\vert\) il est donc suffisant de supposer les fonction \(f_n\) positives.

Soit \(x\in [0,1]\) fixé, posons pour \(k\in\mathbb N\) \[I_k=\left\{\,n\in\mathbb N\ :\ \dfrac{1}{2^{k+1}}< f_n(x)\leq \dfrac{1}{2^{k}}\right\}.\] Soient \(n,p\in I_k\) on a \[\dfrac{1}{2^{2k+2}}< f_n(x)f_p(x)\leq \dfrac{1}{2^{\vert n-p\vert}}\] soit \(\vert n-p\vert\leq 2k+2\) qui implique \(\text{card}(I_k)\leq 2k+2+1\). La série \(\sum_n f_n(x)\) étant à termes positifs on peut sommer par paquets : \[0\leq f(x)=\sum_{n\geq 0} f_n(x)=\sum_{n\geq 0}\sum_{k\in I_n} f_k(x)\leq \sum_{n\geq 0} (2n+3)\cdot\dfrac{1}{2^{n}}=10.\] \(f\) est donc bornée par \(10\) sur \([0,1]\) (bien remarqer que nous n’avons pas utilisé la continuité des applications \(f_n\)).

Montrons qu’en général il n’y a pas convergence uniforme sur \([0,1]\). Pour cela, considérons une suite strictement croissante \(x_0=0<x_1<x_2<\dots <1\) de limite \(1\) et les applications \(f_n\in\mathscr C([0,1])\) définies de la manière suivante :

\(\rightsquigarrow\) \(0\leq f_n\leq 1\),

\(\rightsquigarrow\) \(f_n\equiv 0\) sur \([0,1]\setminus [x_n,x_{n+1}]\) et \(f_n(y_n)=1\)\(y_n =(x_n+x_{n+1})/2\),

\(\rightsquigarrow\) \(f_n\) est affine sur \([x_n, y_n]\) et \([y_n,x_{n+1}]\).

Il n’est pas difficile de vérifier que la suite \((f_n)_n\) ainsi définie vérifie (\(\star\)), en effet les supports des applications \(f_n\) étant mutuellement disjoints \[\vert f_n(x)f_m(x)\vert=\begin{cases} 0&\text{si}\ n\neq m\\ \vert f_n(x)\vert^2\leq 1&\text{si}\ n=m\end{cases} \quad \leq 2^{-\vert n-m\vert},\ \forall\,n,m\in\mathbb N.\] Maintenant, comme (toujours par disjonction des supports) \[f(x)=\sum_{k\geq 0}f_k(x)=f_n(x) \quad\text{où}\quad x\in[x_n,x_{n+1}],\] en particulier \[f(y_n)=1,\quad\forall\,n\in\mathbb N.\] Ainsi \[f(0)=0,\quad \lim_n y_n=1\quad \text{et}\quad \lim_n f(y_n)=1,\] \(f\) est donc discontinue au point \(1\) et la convergence ne peut être uniforme sur \([0,1]\).

Remarque : Dans l’exemple précédent, \(f\) est discontinue au point \(1\) en lequel toutes les applications \(f_n\) s’annulent ; il est en fait assez facile de vérifier qu’en un point \(x\in[0,1]\) où il existe un entier \(n\) tel que \(f_n(x)\neq 0\) la série \(\sum_n f_n\) est normalement convergente sur un voisinage de \(x\) et \(f\) est par suite continue au point \(x\).


Limite d’une suite via les séries entières *

9 novembre 2022 22:30 — Par Patrice Lassère

On considère la suite \((a_n)_{n\in\mathbb N}\) définie par \[a_0=1\quad\rm{et}\quad a_{n+1}=\dfrac{1}{n+1}\sum_{k=0}^n\dfrac{a_k}{n-k+2}.\] Déterminer la limite \[\lim_{n\to+\infty}\sum_{k=0}^n\dfrac{a_k}{2^k}.\]



[ID: 2889] [Date de publication: 9 novembre 2022 22:30] [Catégorie(s): Suites et séries de fonctions, séries entières ] [ Nombre commentaires: 0] [nombre d'éditeurs: 1 ] [Editeur(s): Emmanuel Vieillard-Baron ] [nombre d'auteurs: 1 ] [Auteur(s): Patrice Lassère ]
Accordéon
Titre
Solution
Texte

Limite d’une suite via les séries entières
Par Patrice Lassère le 9 novembre 2022 22:30

Par une recurrence immédiate \(0<a_n\leq 1,\ \forall\,n\in\mathbb N\). Le rayon de convergence de la série génératrice associée \(f(x)=\sum_{n\geq 0}a_n x^n\) est donc supérieur ou égal à \(1\) et on nous demande de calculer \(f(1/2)\). Avec la formule de récurrence \[\begin{aligned} f'(x)&=\sum_{n\geq 0}(n+1)a_{n+1}x^n=\sum_{n\geq 0}\sum_{k=0}^n\dfrac{a_k}{n-k+2}x^n\\ &=\sum_{k\geq 0}a_k x^k\sum_{n\geq k}\dfrac{x^{n-k}}{n-k+2}=f(x)\sum_{m\geq 0}\dfrac{x^m}{m+2}. \end{aligned}\] Par conséquent comme \(f(0)=1\) et \(f(x)>0\) sur \(]0,1[\) : \[\log(f(x))=\log(f(x))-\log(f(0))=\int_0^x\dfrac{f'(t)}{f(t)}dt=\int_0^x \sum_{m\geq 0}\dfrac{t^m}{m+2}dt=\sum_{m\geq 0}\dfrac{x^{m+1}}{(m+1)(m+2)}\] l’échange intégrale/série étant justifié par la normale convergence de la série entière sur \([0,x]\subset [0,1[\) , ainsi \[\begin{aligned} \log(f(x))&=\sum_{m\geq 0}\dfrac{x^{m+1}}{(m+1)(m+2)}=\sum_{m\geq 0}\left( \dfrac{x^{m+1}}{m+1}-\dfrac{x^{m+1}}{m+2}\right) \\ &=1+\left( 1-\dfrac{1}{x}\right) \sum_{m\geq 0}\dfrac{x^{m+1}}{m+1}=1+\left( 1-\dfrac{1}{x}\right)\log\left( \dfrac{1}{1-x}\right) \end{aligned}\] soit \(\log(f(1/2))=1-\log(2)\) et finalement \(f(1/2)=e/2\).


Séries entières et convergence uniforme *

9 novembre 2022 22:30 — Par Patrice Lassère

Pour une série entière \(\sum_n\,a_nz^n\) de rayon de convergence \(R\geq 1\), montrer que les propriétés suivantes sont équivalentes

  1. La série \(\sum_n\,a_nz^n\) converge uniformément sur \(D(0,1)\).

  2. La série \(\sum_n\,a_nz^n\) converge uniformément sur \(\overline{D(0,1)}\).

  3. La série \(\sum_n\,a_nz^n\) converge uniformément sur \(C(0,1)\).



[ID: 2891] [Date de publication: 9 novembre 2022 22:30] [Catégorie(s): Suites et séries de fonctions, séries entières ] [ Nombre commentaires: 0] [nombre d'éditeurs: 1 ] [Editeur(s): Emmanuel Vieillard-Baron ] [nombre d'auteurs: 1 ] [Auteur(s): Patrice Lassère ]
Accordéon
Titre
Solution
Texte

Séries entières et convergence uniforme
Par Patrice Lassère le 9 novembre 2022 22:30
  1. (1) \(\implies\) 2)) : C’est la conséquence d’un corollaire presque immédiat (mais essentiel) du critére de Cauchy uniforme : Soient \(X\) un espace topologique, \(E\) un espace de Banach ; alors toute suite de fonctions \((f_n)_n\subset\mathscr C(X,B)\) qui converge uniformément sur une partie \(Y\subset X\) converge encore uniformément sur \(\overline Y\). Comme justification il suffit de remarquer que par continuité des applications \(f_n\), le critère de Cauchy uniforme \[\forall\,\varepsilon>0,\ \exists\, n_\varepsilon\ :\ m,n\geq n_\varepsilon\ \implies \sup_{x\in Y}\Vert f_n(x)-f_m(x)\Vert_E\leq \varepsilon\] implique \[\forall\,\varepsilon>0,\ \exists\, n_\varepsilon\ :\ m,n\geq n_\varepsilon\ \implies \sup_{x\in \overline{Y}}\Vert f_n(x)-f_m(x)\Vert_E\leq \varepsilon\]

  2. Les implications (2) \(\implies\) )3) et (3) \(\implies\) 2)) sont évidentes.

  3. Il suffit donc par exemple d’établir (3) \(\implies\) 2)) : Par convergence uniforme sur le cercle unité, pour tout \(\varepsilon>0\), il existe \(n_\varepsilon\in\mathbb N\) tel que \(n\geq n_\varepsilon\ \implies \ \vert s_n(e^{i\theta})\vert=\vert\sum_{n_\varepsilon}^na_ke^{ik\theta}\vert<\varepsilon\). On va effectuer une transformation d’Abel : soit \(0\leq r\leq 1\) et \(n\geq n_\varepsilon+1\), pour tout \(\theta\in\mathbb R\) \[\begin{aligned} \left\vert \sum_{n_\varepsilon+1}^n a_kr^k e^{ik\theta}\right\vert &=\left\vert\sum_{n_\varepsilon+1}^n r^k (s_k(e^{i\theta})-s_{k-1}(e^{i\theta}))\right\vert\\ &=\left\vert -r^{n_\varepsilon+1}s_{n_\varepsilon}(e^{i\theta})+r^ns_n(e^{i\theta})+\sum_{n_\varepsilon+1}^n s_k(e^{i\theta})(r^k-r^{k+1})\right\vert\\ &\leq \left\vert r^{n_\varepsilon+1}s_{n_\varepsilon}(e^{i\theta})\right\vert+\left\vert r^ns_n(e^{i\theta})\right\vert+\sum_{n_\varepsilon+1}^n (r^k-r^{k+1})\left\vert s_k(e^{i\theta})\right\vert\\ &\leq \varepsilon\left[ r^{n_\varepsilon+1}+r^n+\sum_{n_\varepsilon+1}^n (r^k-r^{k+1})\right]\\ &\leq 2\varepsilon r^{n_\varepsilon+1}\leq 2\varepsilon \end{aligned}\] Le critère de Cauchy uniforme est donc bien vérifié sur \(\overline{D(0,1)}\), soit 2).

    Remarque : Le candidat à l’agrégation externe peut régler l’implication délicate (3) \(\implies\) 2)) trés simplement en invoquant le principe du maximum.


Convergence uniforme et convergence continue *

9 novembre 2022 22:30 — Par Patrice Lassère

Soit \(A\subset\mathbb R\) une partie non vide (ou plus généralement d’un espace vectoriel normé) et \((f_n)_n\) une suite d’applications de \(A\) dans \(\mathbb K\). On dira que la suite \((f_n)_n\) converge continuement vers \(f\ :\ A\to\mathbb K\) si pour tout \(x\in A\), pour toute suite \((x_n)_n\subset A\) convergente vers \(x\) la suite \((f_n(x_n))_n\) converge vers \(f(x)\).

  1. Montrer que la convergence continue implique la convergence simple.

  2. Soit \((f_n)_n\) une telle suite, \(x\in A\) et \((x_n)_n\) dans \(A\) convergente vers \(x\). Montrer que pour toute sous-suite \((f_{n_k})_k\) \[\lim_{k\to\infty}f_{n_k}(x_k)=f(x).\]

  3. Si \((f_n)_n\) converge continuement vers \(f\) sur \(A\), montrer que \(f\) est continue sur \(A\) (même si les \(f_n\) ne sont pas continues !)

  4. Montrer que toute suite \((f_n)_n\) uniformément convergente sur \(A\) vers une fonction \(f\in\mathscr C(A,\mathbb K)\) converge continuement sur \(A\). La réciproque est-elle vraie ?

  5. Soit \((f_n)_n\) une suite de fonctions définies sur une partie compacte \(K\). Montrer que les propriétés suivantes sont équivalentes.

    -La suite \((f_n)_n\) est uniformément convergente vers \(f\in\mathscr C^0(K).\)

    -La suite \((f_n)_n\) est continuement convergente sur \(K\) vers \(f\).



[ID: 2893] [Date de publication: 9 novembre 2022 22:30] [Catégorie(s): Suites et séries de fonctions, séries entières ] [ Nombre commentaires: 0] [nombre d'éditeurs: 1 ] [Editeur(s): Emmanuel Vieillard-Baron ] [nombre d'auteurs: 1 ] [Auteur(s): Patrice Lassère ]
Accordéon
Titre
Solution
Texte

Convergence uniforme et convergence continue
Par Patrice Lassère le 9 novembre 2022 22:30
  1. Si la suite \((f_n)_n\) converge continuement vers \(f\) s ur \(A\) et si pour \(x\in A\) on considère la suite constante \(x_n=x\) alors : \[f(x)=\lim_{n\to\infty}f_n(x_n)=\lim_{n\to\infty}f_n(x)\] i.e. \((f_n)_n\) est simplement convergente sur \(A\) vers \(f\).

  2. Soit donc \((f_{n_k})_k\) une sous-suite de la suite \((f_n)_n\) et \((x_n)_n\) une suite dans \(A\) convergente vers \(x\in A\). Considérons alors la suite \((y_m)_m\) définie par \[y_m=\begin{cases} x_1\quad &\text{pour}\quad 1\leq m\leq n_1,\\ x_2\quad &\text{pour}\quad n_1< m\leq n_2,\\ \dots\quad & ..........\\ x_k\quad &\text{pour}\quad n_k< m\leq n_{k+1}\\ \dots\quad & .......... \end{cases}\] La suite \((y_m)_m\) est bien entendu encore convergente vers \(x\) et on a donc \[f(x)=\lim_{m\to\infty}f_m(x_m)\quad\implies\quad f(x)=\lim_{k\to\infty}f_{n_k}(y_{n_k})=\lim_{k\to\infty}f_{n_k}(x_k).\]

  3. Avec la première question, si \((f_n)_n\) converge continuement vers \(f\) sur \(A\), elle converge simplement sur \(A\) vers \(f\). Montrons que \(f\) est continue sur \(A\) : soit \(x\in A,\ (x_n)_n\subset A\) une suite convergente vers \(x\). Pour tout \(\varepsilon>0\), par la convergence de \((f_n(x_1))_n\) vers \(f(x_1)\) il existe \(n_1\) (qui à priori dépend de \(x_1\)) tel que \[\vert f_{n_1}(x_1)-f(x_1)\vert\leq \dfrac{\varepsilon}{2}.\] De même, il existe \(n_2>n_1\) tel que \[\vert f_{n_2}(x_2)-f(x_2)\vert\leq \dfrac{\varepsilon}{2}.\] En réitérant ce processus, on construit une suite strictement croissante d’entiers vérifiant \[\vert f_{n_k}(x_k)-f(x_k)\vert\leq \dfrac{\varepsilon}{2}\quad\forall\,k\in\mathbb N.{(1)}\] Mais avec la question précédente \(\lim_{k\to\infty}f_{n_k}(x_k)=f(x)\), si bien qu’il existe aussi un entier \(k_0\) tel que \[\vert f_{n_k}(x_k)-f(x)\vert\leq \dfrac{\varepsilon}{2},\quad\forall\,k\geq k_0.{(2)}\] Finalement \((1)\) et \((2)\) assurent que \[\vert f(x_k)-f(x)\vert \leq \vert f_{n_k}(x_k)-f(x_k)\vert + \vert f_{n_k}(x_k)-f(x)\vert\leq \varepsilon\quad\forall\,k\geq k_0\] et \(f\) est continue au point \(x\), elle est donc continue sur \(A\).

  4. -Supposons maintenant que la suite \((f_n)_n\) converge uniformément sur \(A\) vers une fonction continue \(f\) (les fonctions \(f_n\) n’étant pas continues, l’hypothèse de continuité sur \(f\) est essentielle vu la question précédente) et montrons que la convergence est continue sur \(A\). Soit donc \((x_n)_n\subset A\) une suite convergente vers \(x\in A\). Soit \(\varepsilon>0\), par convergence uniforme sur \(A\) nous avons \[\vert f_n(x_n)-f(x_n)\vert\leq \sup_{y\in A}\vert f_n(y)-f(y)\vert \leq\dfrac{\varepsilon}{2},\quad\forall\,n\geq n_0.{(3)}\] Et par continuité de \(f\) \[\vert f(x_n)-f(x)\vert\leq\dfrac{\varepsilon}{2},\quad\forall\,n\geq n_1.{(4)}\] Ainsi, pour \(n\geq\max\{n_0,n_1\}\), nous avons vu \((3)\) et \((4)\) \[\vert f_n(x_n)-f(x)\vert\leq \vert f_n(x_n)-f(x_n)\vert+\vert f(x_n)-f(x)\vert\leq\dfrac{\varepsilon}{2}\] d’où la convergence continue sur \(A\).

    -La réciproque est fausse. Considérons par exemple \(A=]0,1[\) et \(f_n(x)=x^n\). Il est facile de vérifier que la suite \((f_n)_n\) simplement convergente sur \(]0,1[\) vers la fonction \(f\) identiquement nulle n’est pas uniformément convergente sur \(]0,1[\). Toutefois cette suite converge continuement sur \(]0,1[\) vers \(f\) car pour toute suite \((x_n)_n\subset]0,1[\) convergente vers \(x\in]0,1[\) il existe \(0<a<1\) tel que \(0<x_n<a\) de sorte que \[\vert f_n(x_n)-f(x)\vert=\vert f_n(x_n)\vert\leq a^n\] et donc \[\lim_{n\to\infty}f_n(x_n)=0=f(x).\]

  5. -la condition nécéssaire \((\Rightarrow)\) à été établie lors de la question précédente.

    -Pour la condition suffisante \((\Leftarrow)\), avec la question 2), nous savons déja que la limite \(f\) est continue sur \(K\). Supposons maintenant que notre suite \((f_n)_n\) ne converge pas uniformément sur \(K\) : il existe donc \(\varepsilon_0>0\), une suite \((n_k)_k\) d’entiers et une suite \((x_k)_k\) dans \(K\) tels que \[\forall\,k\in\mathbb N\ :\qquad\vert f_{n_k}(x_k)-f(x_k)\vert\geq\varepsilon_0.{(5)}\] Comme \(K\) est compact on peut supposer (quitte à extraire une sous-suite) que la suite \((x_k)_k\) converge vers \(x\in K\). Avec la question 1), il existe alors \(N_0\in\mathbb N\) tel que \[\vert f_{n_k}(x_k)-f(x)\vert\leq\dfrac{\varepsilon_0}{3},\quad\forall\,n\geq N_0.{(6)}\] Par continuité de \(f\), il existe \(N_1\in\mathbb N\) tel que \[\vert f(x_k)-f(x)\vert\leq\dfrac{\varepsilon_0}{3},\quad\forall\,n\geq N_1,{(7)}\] si bien qu’en combinant \((5),(6)\) et \((7)\) on obtient pour \(n\) assez grand \[\varepsilon_0\leq\vert f_{n_k}(x_k)-f(x_k)\vert\leq \vert f_{n_k}(x_k)-f(x)\vert+ \vert f(x)-f(x_k)\vert\leq\dfrac{2\varepsilon_0}{3}\] ce qui est absurde.


Étude des séries de fonctions \(\sum_{n\geq 0}\,t^nf(t)\) et \(\sum_{n\geq 0}(-1)^nt^nf(t)\) *

9 novembre 2022 22:30 — Par Patrice Lassère

(
[rms] 1993/94 et 1997/98.)

  1. Soit \(f\ :\ [0,1]\to\Bbb R\) une application bornée. Montrer que la série de fonctions \(\sum_{n\geq 0}\,t^nf(t)\) converge uniformément sur \([0,1]\) si et seulement si \(f\) est dérivable en \(t=1\) avec \(f(1)=f'(1)=0\).

  2. Soit \(f\ :\ [0,1]\to\Bbb R\) une application bornée. Étudier la simple puis uniforme convergence de la série \(\sum_{n\geq 0}(-1)^nt^nf(t)\) sur \([0,1[\).



[ID: 2895] [Date de publication: 9 novembre 2022 22:30] [Catégorie(s): Suites et séries de fonctions, séries entières ] [ Nombre commentaires: 0] [nombre d'éditeurs: 1 ] [Editeur(s): Emmanuel Vieillard-Baron ] [nombre d'auteurs: 1 ] [Auteur(s): Patrice Lassère ]
Accordéon
Titre
Solution
Texte

Étude des séries de fonctions \(\sum_{n\geq 0}\,t^nf(t)\) et \(\sum_{n\geq 0}(-1)^nt^nf(t)\)
Par Patrice Lassère le 9 novembre 2022 22:30
  1. Notons pour \(n\in\mathbb N\)

    \[f_n(t)= \begin{cases} [0,1]&\longrightarrow\quad\mathbb R\\ t&\longmapsto \quad t^nf(t) \end{cases}\]

    -Remarquons qu’il y a bien sûr simple convergence sur \([0,1[\) de somme \(f(t)\over1+t\). En outre il y aura convergence en \(t=1\) si, et seulement si, \(f(1)=0\). En résumé la série est simplement convergente sur \([0,1]\) si, et seulement si, \(f(1)=0\) et

    \[\sum_{n\geq 0}t^nf(t)= \begin{cases} {f(t)\over 1+t} &\quad\text{si }\ t\in[0,1[\\ 0&\quad\text{si }\ t=1. \end{cases}\]

    -Supposons \(f(1)=0\) et désignons par \(R_n\) le reste d’ordre \(n\) de la série simplement convergente \(\sum_{n\geq 0}f_n\). On a donc :

    \[\begin{cases} R_n(t)& =\ \sum_{k\geq n+1}t^kf(t)={t^{n+1}f(t)\over 1-t} = -t^{n+1}{f(t)-f(1)\over t-1},\ t\in[0,1[\\ R_n(1) &=\ 0 \end{cases}\]

    \(\rightsquigarrow\quad\) Supposons \(f\) dérivable en \(t=1\) avec \(f'(1)=0\) et soit \(\varepsilon>0\), il existe \(0<\eta<1\) tel que

    \[\forall\,t\in[1-\eta,1[,\quad\left\vert{f(t)-f(1)\over t-1}\right\vert\leq \varepsilon,\]

    de sorte que

    \[\forall\,n\in\mathbb N,\ \forall\,t\in[1-\eta,1[ \quad :\quad \left\vert R_n(t)\right\vert\leq t^{n+1}\varepsilon\leq \varepsilon. {(1)}\]

    D’autre part, l’application \(t\mapsto{f(t)-f(1)\over t-1}\) est bornée sur \([0,1-\eta,]\), il existe donc \(M>0\) tel que

    \[\forall\,t\in[0,1-\eta],\quad \left\vert{f(t)-f(1)\over t-1}\right\vert\leq M\]

    soit

    \[\forall\,n\in\mathbb N,\ \forall\,t\in[0,1-\eta]\quad :\quad \left\vert R_n(t)\right\vert\leq M(1-\eta)^{n+1}.{(2)}\]

    Puisque \(0<\eta<1\), \(\lim_{n\to+\infty}(1-\eta)^{n+1}M=0\) si bien qu’en combinant \((1),\ (2)\) et \(R_n(1)=0\) on peut écrire

    \[\forall\,\varepsilon>0,\ \exists\,N\in\mathbb N,\ \forall\,n\geq N\ \Longrightarrow \sup_{t\in[0,1]}\vert R_n(t)\vert\leq\varepsilon\]

    qui équivaut à l’uniforme convergence sur \([0,1]\) de \(\sum_n f_n\).

    \(\rightsquigarrow\quad\) Réciproquement, supposons la convergence uniforme sur \([0,1]\) et soit \(\varepsilon>0\) fixé. Il existe \(N\in\mathbb N\) tel que

    \[n\geq N\ \Longrightarrow\ \vert R_n(t)\vert \leq\varepsilon\]

    soit

    \[\forall\,t\in[0,1]\ :\ \left\vert{f(t)-f(1)\over t-1}\right\vert\leq\varepsilon t^{-(N+1)} {(3)}\]

    et comme \(\lim_{t\to 1_-}t^{-(N+1)}=1\) il existe \(0<\eta<1\) tel que

    \[1-\eta<t\leq 1 \quad\Longrightarrow\quad 0\leq t^{-(N+1)}\leq 2{(4)}\]

    \((3)\) et \((4)\) nous donnent

    \[\forall\,\varepsilon>0,\ \exists\,\eta\in]0,1[\quad :\quad \left\vert{f(t)-f(1)\over t-1}\right\vert\leq 2\varepsilon\]

    i.e. \(f\) est dérivable en \(t=1\) et \(f'(1)=0\)

    En résumé, la série \(\sum_{n\geq 0}t^nf(t)\) converge uniformément sur \([0,1]\) si, et seulement si \(f\) est dérivable au point \(t=1\) avec \(f(1)=f'(1)=0\).

  2. -Pour \(0\leq t<1\) la série est bien évidement convergente avec \(\sum_{n\geq 0}(-1)^nt^nf(t)={f(t)\over 1+t}\). Pour \(t=1\) la série converge si et seulement si, \(f(1)=0\).

    -Étudions maintenant la convergence uniforme sur \([0,1[\). Le reste de la série est

    \[R_n(t):=\sum_{k\geq n+1}(-1)^kt^kf(t)={(-t)^{n+1}f(t)\over 1+t},\]

    si bien que (\(1\over 1+t\) variant sur \([0,1[\) entre \(1/2\) et \(1\)) la série converge uniformément sur \([0,1[\) si, et seulement si, la suite \((t^{n+1}f(t))_n\) tend uniformément vers zéro sur \([0,1[\). Supposons que \(f\) tende vers \(0\) en \(1\) : pour tout \(\varepsilon>0\), il existe \(0<\eta<1\) tel que

    \[1-\eta<t<1\quad\Longrightarrow \vert f(t)\vert\leq\varepsilon.\]

    Par ailleurs, \(f\) est bornée sur \([0,1-\eta]\) :

    \[\exists\, M>0\ :\quad \vert f(t)\vert\leq M,\quad\forall\,t\in[0,1-\eta]\]

    soit

    \[\vert R_n\vert\leq {\max\{\varepsilon,M(1-\eta)^{n+1}\}\over1+t}\leq 2\varepsilon, \quad\text{pour}\ n\ \text{assez grand}\]

    d’où la convergence uniforme sur \([0,1[\) et donc sur \([0,1]\) si on pose \(f(1)=0\).

    -Supposons maintenant la convergence uniforme sur \([0,1[\), en particulier pour tout \(\varepsilon>0\) il existe un entier \(N\in\mathbb N\) tel que

    \[n>N \quad\Longrightarrow\quad \vert t^{n+1}f(t)\vert\leq \varepsilon,\quad \forall\,t\in[0,1[\]

    et posant \(\eta=1-2^{-{1\over n+1}}\) on a \(\vert f(t)\vert\leq 2\varepsilon,\ \forall\,t\in[1-\eta,1[\), autrement dit

    \[\lim_{t\to1_-}f(t)=0\]

    -En résumé, la série converge uniformément sur \([0,1[\) si, et seulement si \(\lim_{t\to1_-}f(t)=0\). Si par ailleurs \(f(1)=0\) la convergence est alors uniforme sur \([0,1]\).


approximation, convergence uniforme *

9 novembre 2022 22:30 — Par Patrice Lassère

Si une suite de polynômes converge uniformément sur \(\mathbb R\), alors sa limite est un polynôme.



[ID: 2897] [Date de publication: 9 novembre 2022 22:30] [Catégorie(s): Suites et séries de fonctions, séries entières ] [ Nombre commentaires: 0] [nombre d'éditeurs: 1 ] [Editeur(s): Emmanuel Vieillard-Baron ] [nombre d'auteurs: 1 ] [Auteur(s): Patrice Lassère ]
Accordéon
Titre
Solution
Texte

approximation, convergence uniforme
Par Patrice Lassère le 9 novembre 2022 22:30

Soit \((P_n)_n\) une telle suite et \(f\) sa limite. La suite \((P_n)_n\) satisfait donc au critère de Cauchy uniforme sur \(\mathbb R\), en particulier avec \(\varepsilon=1\)

\[\left( \exists N\in\mathbb N\ :\ n\geq N\ \&\ p\in\mathbb N\ \right) \Longrightarrow \left( \sup_{x\in\mathbb R}\vert P_n(x)-P_{n+p}(x)\vert\leq 1 \right)\]

en particulier pour \(p\in\mathbb N\), le polynôme \(P_N-P_{N+p}\) borné sur \(\mathbb R\) est constant

\[\forall\,p\in\mathbb N\ :\ \exists\,C_p\in\mathbb R\ :\ P_N-P_{N+p}=C_p\qquad(\star)\]

et puisque

\[\lim_p P_{N+p}(x)=f(x),\quad\forall\,x\in\mathbb R,\]

en faisant \(x=0\) dans (\(\star\)) on en déduit que la suite \((C_p)_p\) converge

\[\lim_p \left(P_N(0)-P_{N+p}(0)\right)=P_N(0)-f(0)=l=\lim_p C_p.\]

On passe alors à la limite sur \(p\) dans (\(\star\)) pour tout réel \(x\in\mathbb R\) arbitraire :

\[l=\lim_p C_p=\lim_p \left(P_N(x)-P_{N+p}(x)\right)=P_N(x)-f(x)\]

soit

\[f(x)=l+P_N(x),\quad x\in\mathbb R.\quad\]


Une caractérisation de la fonction sinus *

9 novembre 2022 22:30 — Par Patrice Lassère

[rms] (3)-2005/06

Soit \(f\in\mathscr C^\infty(\mathbb R,\mathbb R)\) une application \(2\pi\)-périodique. Si \(f'(0)=1\) et \(\vert f^{(n)}(x)\vert\leq 1,\) pour tout \(x\in\mathbb R\) et \(n\in\mathbb N\) montrer que \(f(x)=\sin(x)\).



[ID: 2899] [Date de publication: 9 novembre 2022 22:30] [Catégorie(s): Suites et séries de fonctions, séries entières ] [ Nombre commentaires: 0] [nombre d'éditeurs: 1 ] [Editeur(s): Emmanuel Vieillard-Baron ] [nombre d'auteurs: 1 ] [Auteur(s): Patrice Lassère ]
Accordéon
Titre
Solution
Texte

Une caractérisation de la fonction sinus
Par Patrice Lassère le 9 novembre 2022 22:30
  1. Le cas \(\mathbf{2}\mathbf{\pi}\)-périodiquePremière étape : \(f\) est de classe \(\mathscr C^\infty\) et \(2\pi\)-périodique on peut donc lui appliquer les relations bien connues reliant les coefficients de Fourier d’une fonction a ceux de ses dérivées : \[\forall\,k\in\mathbb N,\ \forall\,n\in\mathbb Z\quad :\quad c_n(f^{(k)})=(in)^kc_n(f)\] qui impliquent \[\forall\,k\in\mathbb N,\ \forall\,n\in\mathbb Z\quad :\quad \vert c_n(f^{(k)})\vert=\vert n\vert^k\vert c_n(f)\vert.{(\text{$\star$})}\] Soit, vu les hypothèses sur \(f\) et ses dérivées : \[\forall\,k\in\mathbb N,\ \forall\,n\in\mathbb Z\quad :\quad \vert c_n(f^{(k)})\vert =\left\vert\dfrac{1}{2\pi}\int_{-\pi}^{\pi}f^{(k)}(t)e^{-int}dt\right\vert =\left\vert\dfrac{1}{2\pi}\int_{-\pi}^{\pi}f^{(k)}(t)e^{-int}dt\right\vert\leq 1. {(\text{$\star$})}\] Supposons maintenant qu’il existe \(n_0\in\mathbb Z\) tel que \[\vert n_0\vert \geq 2\quad\text{et}\quad \vert c_{n_0}(f)\vert > 0\] alors vu () : \[\lim_{k\to+\infty}\vert c_{n_0}(f^{(k)})\vert=\lim_{k\to+\infty}{n_0}^k\vert c_{n_0}(f)\vert=+\infty\] contredisant \((\text{$\star$})\), ainsi \[c_n(f)=0,\quad\forall \,n\in\mathbb Z\setminus\left\lbrace -1,0,1\right\rbrace\] et la série de Fourier de \(f\) est de la forme \[c_{-1}(f)e^{-ix}+c_0(f)+c_1(f)e^{ix}=a\cos(x)+b\sin(x)+c\quad \text{où }\ a,b,c\in\mathbb R.\] \(f\) étant \(\mathscr C^\infty\) et \(2\pi\)-périodique sur \(\mathbb R\), les théorèmes classiques de convergence nous assurent que \[f(x)=a\cos(x)+b\sin(x)+c,\quad\forall\,x\in\rbrack-\pi,\pi\lbrack.\]

    Seconde étape : Il ne reste plus qu’à exploiter les trois dernières hypothèses \[f'(0)=1,\quad\Vert f\Vert_\infty\leq 1\quad\text{ et }\quad \Vert f'\Vert_\infty\leq 1\] En effet, \(f'(0)=1\) implique \(b=1\) et, au voisinage de zéro \[f'(x)=a\sin(x)-\cos(x)=-1+ax+o(x)\] qui sera, si \(a\ne 0\), strictement plus petit que \(-1\) lorsque \(x\) tendra vers zéro suivant le signe contraire de \(a\) : c’est contraire à l’hypothèse et donc \(a=0\). Regardant au voisinage de \(\frac{\pi}{2}\), on obtient \(c=0\), soit finalement \[f(x)=\sin(x)\quad\forall\, x\in\mathbb R.\] Q.E.D.

    Sans la \(2\pi\)-périodicité, on pourrai bien sûr envisager de réitérer le même raisonnement sur la fonction \(g\) \(2\pi\)-périodique sur \(\mathbb R\) et égale à \(f\) sur \(]-\pi,\pi]\) pour en déduire que \(g\) puis \(f\) (car \(f\) est clairement développable en série entière sur \(\mathbb R\) et égale à \(g\) sur \(]-\pi,\pi]\) ) coïncide avec la fonction sinus sur \(]-\pi,\pi]\) ; mais malheureusement une obstruction apparait : \(g\) n’étant même plus continue (à priori) en les points de \(\pi\mathbb Z\), les formules \(\forall\,k\in\mathbb N,\ \forall\,n\in\mathbb Z\ :\ c_n(g^{(k)})=(in)^kc_n(g)\) ne sont plus valables. En effet elle s’obtiennent aprés \(k\) intégrations par parties dans \(c_n(g^{(k)})\), les termes entre crochets disparaissant grâce au caractère continu (et la \(2\pi\) périodicité) de \(g\) en \(-\pi\) et \(\pi\) ; de ce fait, dans notre cas, la non continuité de \(g\) et à fortiori de ses dérivées en ces points va faire apparaitre à droite un polynôme en \(n\) qui (semble) rendre vain tout espoir de généralisation par cette méthode....

  2. Le cas général On va proposer deux solutions (ces deux solutions sont tirées du volume 116-3 de la RMS, [rms]), l’une s’appuyant sur la théorie des fonctions holomorphes, l’autre sur l’analyse fonctionnelle. Commencons par fixer quelques notations communes aux deux solutions.

    Pour toute fonction bornée sur \(\mathbb R\), on pose \(\Vert f\Vert=\sup_{x\in\mathbb R}\vert f(x)\vert.\) L’espace vectoriel \[\mathscr E_\mathbb C:=\{f\in\mathscr C^\infty(\mathbb R,\mathbb C)\ :\ \exists\,c>0\ \forall\,n\in\mathbb N,\ \forall\,x\in\mathbb R\quad \vert f^{(n)}(x)\vert\leq c\}\] (on définit de même l’espace réel \(\mathscr E_\mathbb C\)) sera normé par \(\displaystyle N(f)=\sup_{n\in\mathbb N}\Vert f^{(n)}\Vert\) ; on désignera par \(\mathscr E_\mathbb R\) l’espace vectoriel réel \(\mathscr E_\mathbb R\subset\mathscr E_\mathbb C\) des applications à valeurs réelles. Enfin, \(\mathscr B\) désignera l’espace des fonctions entières \(f\) vérifiant \[\exists\, c>0\ :\quad \forall\,z\in\mathbb C, \quad \vert f(z)\vert\leq ce^{\vert\text{Im}(z)\vert},\] il est normé par \(\displaystyle N_\mathscr B(f)=\sup_{z\in\mathbb C}\vert f(z)\vert e^{-\vert\text{Im}(z)\vert}\).

    Notre objectif est alors le suivant

    Théorème : La solution dans \(f\in\mathscr E_\mathbb R\) de \(N(f)\leq 1\) & \(f'(0)=1\) est \(f=\sin\).

    Résultats préliminaires :

    \(\rightsquigarrow\)Pour toute application \(f\in \mathscr E\), il existe une unique fonction entière \(\tilde f\) dont la restriction à \(\mathbb R\) est \(f\). De plus, \(\tilde f\in\mathscr B\) et \(\quad N_{\mathscr B}(\tilde f)\leq N(f)\).

    \(\rightsquigarrow\)Pour tout \(t\in\mathbb R\) (resp. \(\tau\in\mathbb C\)) et \(f\in\mathscr E\) (resp. \(\mathscr B\)) les translatés \(x\mapsto f_t(x)=f(x+t)\) (resp. \(f_\tau\)) restent dans \(\mathscr E\) (resp. \(\mathscr B\)).

    \(\rightsquigarrow\)De toute suite \((f_n)_n\) bornée de \((\mathscr E,N)\) il existe une application \(f\in\mathscr E\) et une sous-suite \((f_{n_k})_k\) telles que pour tout entier \(j\in\mathbb N\) la suite \((f^{(j)}_{n_k})_k\) converge uniformément sur tout compact de \(\mathbb R\) vers \(f^{(j)}\) (i.e. converge dans1 \(\mathscr C^\infty(\mathbb R)\)).

    Preuve : Les hypothèses sur \(f\) assurent qu’elle est la restriction à la droite réelle de la série entière \[\tilde f(z):=\sum_{n\geq 0}\dfrac{f^{(n)}(0)}{n!}z^n\] de rayon de convergence infini. En outre pour tout \(x+iy\in\mathbb C\) \[\vert \tilde f(x+iy)\vert\leq \sum_{n\geq 0}\left\vert\dfrac{f^{(n)}(x)}{n!}(iy)^n\right\vert\leq\sum_{n\geq 0}\dfrac{\vert y\vert}{n!}=e^{\vert y\vert}N(f)\] soit \(f\in\mathscr B\) et \(N_{\mathscr B}(\tilde f)\leq N(f)\).

    La seconde assertion est immédiate et la troisième se déduit du fait que toute partie bornée de \((\mathscr E, N)\) est bornée dans l’espace de Montel \(\mathscr C^\infty(\mathbb R)\) .\(\square\)

    Première solution : Pour \(f\in\mathscr E_\mathbb R\) posons \[Q(f)=f(0)^2+f'(0)^2,\quad R(f)=\Vert f^2+f'^2\Vert.\] Il est clair que \(Q(f)\leq R(f)\leq 2N(f)^2,\quad \forall\,f\in\mathscr E_\mathbb R,\) ce qui justifie la définition \[\rho:=\sup_{f\in\mathscr E_\mathbb R,\ N(f)\leq 1}R(f).\] La démonstration repose sur la détermination de \(\rho\) et l’étude de \(X:=\{ f\in\mathscr E_\mathbb R\ :\ N(f)\leq 1\ \text{ et }\ Q(f)=\rho\,\}\), elle se divise en trois étapes.

    On peut déja remarquer que \(\rho\geq 1\) car l’application \(x\mapsto \sin(x)\) est dans \(\mathscr E_\mathbb R\) .

    L’ensemble \(X\) est non vide.

    En effet, considérons une suite \((f_k)_k\) d’éléments de \(\mathscr E_\mathbb R\) telle que \[N(f_k)\leq 1,\ \forall\,k\in\mathbb N\ \text{ et }\ \lim_{n\to\infty}R(f_k)=\rho.\] Quitte à remplacer chaque \(f_k\) par une de ses translatés (via le lemme, mais il faut être tout de même délicat car le sup peut être atteint à l’infini...) on peut supposer que \(\lim_k Q(f_k)=\rho\). La troisième assertion du lemme assure alors de l’existence d’une sous-suite \((f_{n_k})_k\) et d’une application \(f\in\mathscr E_\mathbb R\) telles que pour tout entier \(j\in\mathbb N\) la suite \((f_{n_k}^{(j)})_k\) converge uniformément sur tout compact de \(\mathbb R\) vers \(f^{(j)}\). Clairement, \(f\in X\) qui est non vide. \(\square\)

    Soit \(f\in X\). Si \(f'(0)\neq 0\), alors \(f''(0)=-f'(0)\) et \(f'\in X\).

    Vu la définition de \(\rho\) et de \(X\), pour toute \(f\in X\), la fonction \(g:=f^2+f'^2\) admet un maximum local en \(x=0\), soit \(g'(0)=0\) et le résultat suit.\(\square\)

    \(\rho=1.\)

    Supposons par l’absurde que \(\rho>1\). Dans ce cas, pour tout \(f\in X\) : \(f(0)f'(0)\neq 0\) (ne pas oublier que dans ce cas \(Q(f)=f^2(0)+f'^2(0)>1\) et \(\max\{\vert f(0)\vert,\vert f'(0)\vert\}\leq 1\)...) ; on peut donc, en invoquant la seconde étape, en déduire que \[\forall\,f\in X,\ n\in\mathbb N\ :\quad f^{(n)}\in X\quad \text{et}\quad f^{n+2}(0)=-f^{(n)}(0).\] Toutes les applications \(f\) étant développables en série entière, \(X\) est donc inclu dans le plan engendré par les fonctions \(\sin\) et \(\cos\), mais sur ce plan \(Q\) et \(N^2\) coïncident ce qui contredit l’assertion \(\rho>1\).\(\square\)

    Preuve du théorème :Considérons \[Y=\{\,f\in\mathscr E_\mathbb R\ :\quad N(f)\leq 1\ \text{ et }\ f'(0)=1\,\}.\] Vu ce qui précède, \(Y\subset X\) et \(\forall\,f_in X\ :\ f(0)=0\). Pour conclure, il suffit de montrer que \[(f_in Y)\ \Longrightarrow\ (-f''\in Y),\] en effet, si tel est le cas on aura \[\forall\,f\in Y,\ \forall\,n\in\mathbb N\ :\quad f^{(2n)}(0)=0,\ f^{(2n+1)}(0)=(-1)^n\] soit (\(f\) étant développable en série entière) \(f=\sin\).

    Soit donc \(f_inY\). En considérant à nouveau \(g=f^2+f'^2\), comme \(g\) présente à l’origine un maximum local, donc \(g''(0)\leq 0\) qui implique (pas clair encore...) \(f'''(0)\leq -1\) ; et comme \(N(f)\leq 1\) ceci implique \(f'''(0)=-1\) i.e. \(-f''\in Y\), ce qu’il restait à établir. \(\square\)

    Seconde solution :


  1. [watop], [waint].1  Pour la topologie usuelle de \(\mathscr C^\infty(\mathbb R)\) voir l’exercice ?? ou

Accordéon
Titre
Solution
Texte

Approximations uniforme de la valeur absolue sur \([-1,1]\)
Par Patrice Lassère le 9 novembre 2022 22:30
  1. \(x\in[-1,1]\), on pose donc \(x=\cos(t),\ t\in[-\pi,\pi]\). La fonction \(g(t)=f(\cos(t))=\vert\cos(t)\vert\) est \(2\pi\)-périodique continue et \(C^1\) par morceaux : elle est donc développable en série de Fourier et sa série de Fourier converge uniformément sur \(\mathbb R\). Par parité

    \[\vert\cos(t)\vert =\lim_{N\to\infty}\left(\sum_{n=1}^Na_n\cos(nt) +{a_0\over 2}\right) \quad\text{ uniformément en } t\in[-\pi,\pi].\]

    Il est alors facile de vérifier que pour tout \(n\in\mathbb N,\ \cos(nt)\) est un polynôme en \(\cos(t)\) (écrire \(\cos(nt)=re\left(\left(\cos(t)+i\sin(t)\right)^n\right)\) et dans ce dernier terme les sinus apparaissent sous une puissance paire, il ne reste plus qu’à écrire \(\sin^{2k}(t)=\left(1-\cos^2(t)\right)^k\)...). Ainsi \(\vert\cos(t)\vert\) est limite uniforme sur \([-\pi,\pi]\) de polynômes en \(\cos(t)\), il ne reste plus qu’à remplacer \(\cos(t)\) par \(x\) pour conclure.

  2. On peut obtenir la formule \((\bigstar)\) avec les séries entières en écrivant \((1+x)^\alpha=\exp\left(\alpha\log(1+x)\right)\), on peut tout aussi bien le faire en montrant dans la formule de Taylor-Lagrange appliquée à \((1+x)^\alpha\) à l’ordre \(N\), que le reste \(r_N(x)\) tend vers zéro lorsque \(N\) tend vers l’infini et ceci pour tout \(x\in]-1,1[\) en effet

    \[\begin{aligned} (1+x)^\alpha&=1+\sum_{n=1}^N {{\alpha(\alpha-1)\dots(\alpha-n+1)}\over{n!}}x^n +{{\alpha(\alpha-1)\dots(\alpha-N)}\over{(N+1)!}}x^{N+1}(1+x\theta_x)^{\alpha-N-1}\\ &=1+\sum_{n=1}^N {{\alpha(\alpha-1)\dots(\alpha-n+1)}\over{n!}}x^n+r_N(x) \end{aligned}\]

    \(0<\theta_x<1\). Or, pour tout \(x\in]-1,1[\)

    \[\lim_{N\to\infty} {{\alpha(\alpha-1)\dots(\alpha-N)}\over{(N+1)!}}x^{N+1}=0\]

    de sorte que pour s’assurer que \(\lim_N r_N(x)=0\), il suffit de montrer que la suite \(\left((1+x\theta_x)^{\alpha-N-1}\right)_N\) est bornée. Et ceci lorsque \(x\in[0,1[\) résulte des inégalités

    \[\begin{cases} 1\leq (1+x\theta_x)^\alpha \leq 2^\alpha\quad&\text{si }\alpha\geq 0 \\ 2^\alpha\leq (1+x)^\alpha\leq (1+x\theta_x)^\alpha\leq 1\quad&\text{si }\alpha<0\\ (1+x\theta_x)^{-N}\leq 1 \end{cases}\]

    d’où \((\bigstar)\) pour \(x\in[0,1[\) et donc sur \(]-1,1[\) puisque le domaine de convergence d’une série entière est toujours une boule.

    Pour obtenir \((\text{$\star$})\) il n’y a plus qu’à écrire

    \[\vert x\vert=\sqrt{1-(1-x^2)}\]

    et appliquer \((\bigstar)\) avec \(\alpha=1/2\).

  3. La troisième question est plus classique, consultez votre livre de chevet favori.

    Remarques : -On peut s’étonner d’un tel engoument pour approcher la valeur absolue : c’est un tic historique probablement du à la preuve trés ingénieuse que le jeune H.Lebesgue (23 ans) donne en 1898 du célèbre théorème de Weierstrass (toute fonction continue sur un intervalle \([a,b]\) est limite uniforme de polynômes), pour cela, il commence par observer qu’il est facile d’approcher une fonction continue par une application continue affine par morceaux, qu’un tel objet est combinaison linéaire de translations de \(\vert x\vert\) ; les polynômes étant invariant par translation, il suffit donc d’approcher uniformément \(\vert x\vert\) sur tout voisinage de l’origine. Ce que fit Lebesgue en exhibant justement celle de la seconde série (ce fut sa première publication...).

    -Dans la seconde question, la théorie des séries entières assure la convergence uniforme (et même normale) seulement sur tout \([-a,a]\subset]-1,1[\). En fait comme pour les autres exemples, la convergence est uniforme sur \([-1,1]\), elle résulte de......................................


\(f(x)=\sum_{m=0}^\infty e^{-m}\cos(m^2x)\) n’est pas développable en série entière *

9 novembre 2022 22:30 — Par Patrice Lassère

Exemple d’une fonction \(\mathscr C^\infty\) sur \(\mathbb R\) dont la série de Taylor en \(x=0\) admet un rayon de convergence nul :

Soit

\[f(x)=\sum_{m=0}^\infty e^{-m}\cos(m^2x)=\sum_{m=0}^\infty f_m(x),\]

montrer que \(f\in \mathscr C^\infty(\mathbb R)\) mais n’est pas développable en série entière à l’origine.



[ID: 2903] [Date de publication: 9 novembre 2022 22:30] [Catégorie(s): Suites et séries de fonctions, séries entières ] [ Nombre commentaires: 0] [nombre d'éditeurs: 1 ] [Editeur(s): Emmanuel Vieillard-Baron ] [nombre d'auteurs: 1 ] [Auteur(s): Patrice Lassère ]
Accordéon
Titre
Solution
Texte

\(f(x)=\sum_{m=0}^\infty e^{-m}\cos(m^2x)\) n’est pas développable en série entière
Par Patrice Lassère le 9 novembre 2022 22:30

Que \(f\) soit de classe \(\mathscr C^\infty\) et que l’on puisse dériver sous la somme relève du théorème de Weierstrass, en effet les séries des dérivées de tout ordre sont normalement convergente sur \(\mathbb R\) car l’exponentielle l’emporte sur la puissance et les fonctions \(\sin\) et \(\cos\) sont bornées sur \(\mathbb R\)...

\[f\in \mathscr C^\infty(\mathbb R)\quad\text{et}\quad \forall\,k\in\mathbb N,\,x\in\mathbb R\ :\ f^{(k)}(x)= \sum_{m\geq 0}f^{(k)}_m(x)\]

en particulier

\[f^{(2n)}(0)=(-1)^n\sum_{m\geq 0}e^{-m}(m^2)^{2n}.\]

Donc, sous réserve de convergence, la série de Taylor de \(f\) à l’origine est donnée par (les dérivées impaires de \(f\) sont nulles)

\[\sum_{n \geq 0}{{f^{(n)}(0)}\over n!}x^n = \sum_{n \geq 0}{{f^{(2n)}(0)}\over (2n)!}x^{2n} =\sum_{n\geq 0} {{(-1)^nx^{2n}}\over (2n)!} \left(\sum_{m\geq 0}e^{-m}(m^2)^{2n}\right).\]

en particulier

\[\begin{aligned} {{\vert x\vert^{2n}}\over{(2n)!}}\vert f^{(2n)}(0)\vert &= {{\vert x\vert^{2n}}\over{(2n)!}}\sum_{m\geq 0}e^{-m}m^{4n}\\ &\geq \left({{\vert x\vert}\over{2n}}\right)^{2n} \sum_{m\geq 0}e^{-m}m^{4n}\\ &\geq \left({{\vert x\vert m^2}\over{2n}}\right)^{2n}e^{-m},\quad\forall\,m\in\mathbb N. \end{aligned}\]

et si on prend \(m=2n\)

\[{{\vert x\vert^{2n}}\over{(2n)!}}\vert f^{(2n)}(0)\vert \geq \left({{\vert x\vert 2n}\over{e}}\right)^{2n} \geq 1 \quad\text{ dès que }\quad n>{e\over 2\vert x\vert}\]

i.e. pour tout \(x\in\mathbb R\), le terme général de la série de Taylor de \(f\) à l’origine ne tend pas vers zéro dès que \(x\not=0\) : la série diverge grossièrement, le rayon de convergence est donc nul.

voir aussi l’exercice ??? pour l’exemple archi-classique d’une fonction \(\mathscr C^{\infty}\) sur \(\mathbb R\) non développable en série entière à l’origine à série de Taylor à l’origine convergente sur \(\mathbb R\).


Développement en série de Fourier de \(f(x)={{1+\cos(x)}\over{4-2\cos(x)}}\), série entière *

9 novembre 2022 22:30 — Par Patrice Lassère

(
[rms],1997/98). Développer en série de Fourier la fonction \(\displaystyle\qquad f(x)={{1+\cos(x)}\over{4-2\cos(x)}}\).



[ID: 2905] [Date de publication: 9 novembre 2022 22:30] [Catégorie(s): Suites et séries de fonctions, séries entières ] [ Nombre commentaires: 0] [nombre d'éditeurs: 1 ] [Editeur(s): Emmanuel Vieillard-Baron ] [nombre d'auteurs: 1 ] [Auteur(s): Patrice Lassère ]
Accordéon
Titre
Solution
Texte

Développement en série de Fourier de \(f(x)={{1+\cos(x)}\over{4-2\cos(x)}}\), série entière
Par Patrice Lassère le 9 novembre 2022 22:30

\(f\) est clairement \(\mathscr C^\infty\) sur \(\mathbb R\), paire, \(2\pi\)-périodique, elle admet donc un développement en série de Fourier de la forme \(\displaystyle f(x)=\sum_{n=0}^\infty a_n\cos(nx)\) et la convergence est uniforme sur \(\mathbb R\) (Dirichlet).

Il est sage de se persuader qu’un calcul direct des coefficients de Fourier \[a_n={1\over 2\pi}\int_0^{2\pi}{{1+\cos(x)}\over{4-2\cos(x)}}\cos(nx)dx\] est plus qu’incertain, voire déconseillé, voici donc deux méthodes qui donnent ce développement par des chemins détournés.

  1. Première méthode : \[f(x)={{1+\cos(x)}\over{4-2\cos(x)}}={{2+e^{ix}+e^ {-ix}}\over{2(4-e^{ix}-e^{-ix})}}= {{2e^{ix}+e^{2ix}+1}\over{2(4e^{ix}-e^{2ix}-1)}}=F(e^{ix})\text{ où }F(X)=-{{X^2+2X+1}\over{2(X^2-4X+1)}}\]

    Les pôles de \(F\) sont \(\alpha=2-\sqrt{3}\) et \(\alpha^{-1}\) et on peut (après avoir décomposé en éléments simples ) développer \(F\) en puissance relatives de \(X\), (on retrouve le développement en série de Laurent de la fonction méromorphe \(F\) sur \(\mathbb C\) sur la couronne \(C(0,\alpha,\alpha^{-1})\) pour ceux qui s’en souviennent)

    \[\begin{aligned}F(X)& = -{1\over 2}\left(1-{{\alpha\sqrt{3}}\over{X(1-{\alpha\over X})}}-{{\sqrt{3}}\over{1-\alpha X}}\right)\\ & = -{1\over 2}+{{\sqrt 3}\over 2}\sum_{n\in\mathbb Z}\alpha^{\vert n\vert}X^n,\qquad \alpha<\vert X\vert<\alpha^{-1} \end{aligned}\]

    le cercle unité étant clairement inclus dans la couronne \(\{\alpha<\vert X\vert<\alpha^{-1}\}\) on peut faire \(X=e^{ix}\) dans le développement précédent

    \[f(x)={{1+\cos(x)}\over{4-2\cos(x)}}=F(e^{ix})= -{1\over 2}+{{\sqrt 3}\over 2}\sum_{n\in\mathbb Z}\alpha^{\vert n\vert}e^{inx}={{\sqrt{3}-1}\over 2}+{\sqrt{3}\over 2}\sum_{n\geq 1}\sqrt{3}\left(2-\sqrt{2}\right)^n\cos(nx),\ \forall\,x\in\mathbb R.\]

    Nous avons donc trouvé un développement en série trigonométrique de \(f\) sur \(\mathbb R\), visiblement normallement convergente sur \(\mathbb R\) : c’est le développement en série de Fourier de \(f\) (la convergence uniforme sur un intervalle de longueur \(2\pi\) permet de s’en assurer (échange justifié de somme et d’intégrale))

  2. Seconde méthode : nous savons que \(\displaystyle f(x)=\sum_{n=0}^\infty a_n\cos(nx)={{1+\cos(x)}\over{4-2\cos(x)}}\) qu’on peut aussi écrire

    \[\left( 4-2\cos(x)\right)\sum_{n=0}^\infty a_n\cos(nx)=1+\cos(x)\]

    la convergence étant normale sur \(\mathbb R\) en réordonnant, on trouve

    \[\left( 4a_0-a_1-1\right)+\left(4a_1-a_0-a_2-1\right)\cos(x)+\sum_{n\geq 2}\left(4a_n-a_{n-1}-a_{n+1}\right)\cos(nx)=0\]

    cette série trigonométrique étant normalement convergente sur \(\mathbb R\) ses coefficients sont nuls (c’est du cours) donc

    \[(\bigstar)\qquad \begin{cases} 4a_0-a_1&=1\\ 4a_1-a_0-a_2&=1\\ 4a_n-a_{n-1}-a_{n+1}&=0,\quad \forall\,n\geq 2. \end{cases}\]

    on retrouve un système classique à résoudre : son équation caractéristique est \(r^2-4r+1=0\) la solution générale est \(a_n=\lambda \left(2+\sqrt 3\right)^n+\mu\left(2-\sqrt 3\right)^n\). Comme \(\lim_n a_n=0\), \(\lambda\) est nul et \(a_n=a_1\left(2-\sqrt 3\right)^n,\,n\geq 1\), enfin avec les deux premières équations de \((\bigstar)\) il vient \(a_0={{\sqrt 3-1}\over 2},\ a_1=\sqrt{3}\left(2-\sqrt{3}\right)\) et finalement

    \[f(x)={{\sqrt{3}-1}\over 2}+{\sqrt{3}\over 2}\sum_{n\geq 1}\sqrt{3}\left(2-\sqrt{2}\right)^n\cos(nx).\]


Inégalité de Bernstein via les séries de Fourier *

9 novembre 2022 22:30 — Par Patrice Lassère

A.Pommellet,
[rms]-6, 1992/93. On appelle polynôme trigonométrique toute application de \(\mathbb R\) dans \(\mathbb C\) de la forme \(\displaystyle \quad P(t)= \sum_{k= -n}^n\,a_ke^{ikt},\ a_k\in\mathbb C\) ; et toute application du type \(\displaystyle \quad P(t)= \sum_{k= -n}^n\,a_ke^{i\lambda_kt}\), avec \(\ a_k\in\mathbb C, \lambda_k\in\mathbb R\) est un polynôme trigonométrique généralisé.

L’objectif de cet exercice est d’établir à l’aide des séries de Fourier la célébre inégalité suivante, due au mathématicien russe Bernstein :

\[\Vert P'\Vert_\infty\leq n\,\Vert P\Vert_\infty\quad \text{ pour tout polynôme trigonométrique }\ P.{(\bigstar)}\]

Pour tout polynôme trigonométrique généralisé, on pose \(\displaystyle\Lambda:= \max_{-n\leq k\leq n}\vert \lambda_k\vert\), on va montrer que

\[\Vert P'\Vert_\infty\leq \Lambda\,\Vert P\Vert_\infty \quad \text { pour tout polynôme trigonométrique généralisé }\ P.{(\text{$\star$})}\]

  1. ] Montrer que \((\text{$\star$})\ \Longrightarrow\ (\bigstar)\).

  2. Montrer que pour établir \((\text{$\star$})\) on peut toujours supposer que \(\Lambda= {\pi\over 2}\).

  3. Soit \(\Psi\) la fonction numérique qui vaut \(\ t\) sur \([-{\pi\over 2},{\pi\over 2}]\), \(\pi-t\) sur \([{\pi\over 2},{3\pi\over 2}]\) et qui est prolongée par \(2\pi\)-périodicité sur \(\mathbb R\) tout entier. Montrer que

    \[\Psi(t)= \sum_{l\geq 0}{{4 (-1)^l}\over{\pi(2l+1)^2}}\sin\left((2l+1)t\right), \quad\forall\,t\in\mathbb R.\]

  4. Montrer que \(\displaystyle P'(t)= i\sum_{k= -n}^na_k\Psi(\lambda_k)e {i\lambda_k t}\).

  5. En déduire que \(\quad\displaystyle P'(t)= {2\over \pi}\sum_{l= 0}^\infty {{(-1)^l}\over{(2l+1)^2}} \left( \,\sum_{k= -n}^n\,a_ke^{i(t+2l+1)\lambda_k}-\sum_{k= -n}^n a_ke^{i(t-2l-1)\lambda_k} \right)\)

  6. Puis \(\displaystyle\quad\Vert P'\Vert_\infty\leq {4\over\pi}\left(\sum_{l= 0}^\infty{1\over (2l+1)^2}\right)\Vert P\Vert_\infty\) et conclure.



[ID: 2907] [Date de publication: 9 novembre 2022 22:30] [Catégorie(s): Suites et séries de fonctions, séries entières ] [ Nombre commentaires: 0] [nombre d'éditeurs: 1 ] [Editeur(s): Emmanuel Vieillard-Baron ] [nombre d'auteurs: 1 ] [Auteur(s): Patrice Lassère ]
Accordéon
Titre
Solution
Texte

Inégalité de Bernstein via les séries de Fourier
Par Patrice Lassère le 9 novembre 2022 22:30
  1. C’est clair, car dans ce cas : \(\Lambda= \max\{0,1\dots,n\}= n\).

  2. Si \(\Lambda= 0\), \(P\) est constant et on a même égalité. Si \(\Lambda>0\) est différent de \(\pi/2\) on fait le changement de variable \(u= {2\over\pi\Lambda}t\) et

    \[P(t) = \sum_{k= -n}^n\,a_ke^{i\lambda_kt} = \sum_{k= -n}^n\,a_ke^{i{\pi\lambda_k\over2\Lambda}{2\Lambda\over\pi}t} = \sum_{k= -n}^n\,a_ke^{i{\pi\lambda_k\over 2\Lambda}u} = \sum_{k= -n}^n\,a_ke^{i\tilde\lambda_k u} = \tilde P(u)\]

    on a donc : \(\tilde P (x)= P({\pi x\over 2\Lambda})\) sur \(\mathbb R\), et \(\tilde\Lambda = {\pi\over 2}\) soit

    \[\Vert P\Vert_\infty= \Vert \tilde P\Vert_\infty,\quad\text{ et }\quad {\pi\over 2\Lambda}\Vert P'\Vert_\infty= \Vert\tilde P'\Vert_\infty\]

    (on a ici implicitement utilisé la relation évidente \(\widetilde{P'}= (\tilde P)'\)). Supposons maintenant \((\text{$\star$})\) vraie si \(\Lambda= {\pi\over 2}\), pour tout polynôme trigonométrique généralisé on aura

    \[\Vert\tilde P'\Vert_\infty\leq {\pi\over 2} \Vert \tilde P\Vert_\infty,\quad\text{ soit }\quad {\pi\over 2\Lambda}\Vert P'\Vert_\infty \leq {\pi\over 2} \Vert P\Vert_\infty\]

    i.e.

    \[\Vert P'\Vert_\infty\leq \Lambda\,\Vert P\Vert_\infty{(\text{$\star$})}\]

    Il ne reste donc plus qu’à établir \((\text{$\star$})\) pour tout polynôme trigonométrique généralisé vérifiant \(\Lambda= {\pi\over 2}\).

  3. \(\Psi\) est continue, \(2\pi\) périodique et \(C^1\) par morceaux, le théorème de Dirichlet nous assure que la série de Fourier de \(\Psi\) converge normalement sur \(\mathbb R\) vers \(\Psi\). Et un calcul élémentaire nous donne \[\Psi(t) = \sum_{l\geq 0}{{4(-1)^l}\over{\pi(2l+1)^2}}\sin\left((2l+1)t\right), \quad\forall\,t\in\mathbb R.\]

  4. Soit donc \(\displaystyle \quad P(t)= \sum_{k= -n}^n\,a_ke^{i\lambda_kt}\) un polynôme trigonométrique généralisé vérifiant \(\Lambda= {\pi\over 2}\). La grande astuce consiste à remarquer qu’alors

    \[\left( \Lambda= {\pi\over 2}\right) \quad\Longrightarrow\quad \left( \lambda_k= \Psi(\lambda_k), \quad -n\leq k\leq n \right) \quad\text{puisque sur }\quad[-{\pi\over2},{\pi\over 2}]\ :\ \Psi(t)= t\] on a alors : \[\begin{aligned} \left\vert P'(t)\right\vert &= \left\vert\sum_{k= -n}^n\,a_k i\lambda_ke^{i\lambda_kt}\right\vert\\ &= \left\vert\sum_{k= -n}^n\,ia_k \Psi(\lambda_k)e^{i\lambda_kt}\right\vert\\ &= \left\vert\sum_{k= -n}^n\,ia_k \left(\sum_{l\geq 0}{{4(-1)^l}\over{\pi(2l+1)^2}}\sin\left((2l+1)\lambda_k\right)\right)e^{i\lambda_kt} \right\vert\\ &= \left\vert \sum_{l\geq 0}\sum_{k= -n}^n{2(-1)^l \over \pi(2l+1)^2 }a_k \left( e^{i(t+2l+1)\lambda_k}-e^{i(t-2l-1)\lambda_k} \right) \right\vert\\ &\leq\sum_{l\geq 0}{2\over \pi(2l+1)^2}\left(\left\vert P\left(t+2l+1\right)\left\vert+\right\vert P\left(t-2l-1\right)\right\vert\right)\\ &\leq {4\Vert P\Vert_\infty \over\pi}\sum_{l\geq 0}{1\over (2l+1)^2}\\ &= {\pi\over 2}\Vert P\Vert_\infty,\quad\forall \,t\in\mathbb R, \end{aligned}\]

    (dans la dernière inégalité on a utilisé \(\sum{1\over (2l+1)^2}= {{\pi^2}\over 8}\) qu’on déduit de \(\sum{1\over l^2}= {{\pi^2}\over 6}\)...) soit \(\Vert P'\Vert_\infty\leq {\pi\over 2}\Vert P\Vert_\infty\) d’où \((\text{$\star$})\) si \(\Lambda= {\pi\over 2}\), et d’où \((\text{$\star$})\) pour tout polynôme trigonométrique généralisé vu la deuxième question , d’où l’inégalité de Bernstein par la première question .


Accordéon
Titre
Solution
Texte

Une fonction continue non dérivable à l’origine mais développable en série de Fourier (1)
Par Patrice Lassère le 9 novembre 2022 22:30
  1. Non car en les points \(2k\pi,\ (k\in\mathbb Z\)) \(f\) n’est pas dérivable et n’admet pas de dérivée à droite et à gauche (ailleurs \(f\) est continue et \(\mathscr C^\infty\) sauf en les points \((2k+1)\pi, k\in\mathbb Z\) ou \(f\) n’est pas dérivable mais admet une dérivée à droite et à gauche).

  2. C’est une bestiale intégration par parties faisant apparaître une intégrale généralisée (avec la régle habituelle qu’elle marche si deux termes parmi les trois existent : en zéro pas problème — faire un DL—) il en résulte (le premier terme à droite tend vers zéro car majoré par \(1/2\vert x\vert\) et \({{1-\cos(t^2)}\over{2t^2}}\) est intégrable en l’infini puisque majoré en module par \(1/\vert x\vert^2\) ) que \(\lim_{x\to \infty}G(x)\) existe et vaut \(\int_0^\infty {{1-\cos(t^2)}\over{2t^2}}dt= C>0.\)

  3. On a : \[\begin{aligned}a_n= {2\over\pi}\int_0^\pi\sqrt t \cos(nt)dt &= {2\over\pi}\int_0^{\sqrt{n\pi}} {u\over\sqrt n}\cos(u^2) {2u\,du\over n} \quad(\text{avec le changement }u= \sqrt{nt})\\ &= {4\over\pi n^{3/2}}\int_0^{\sqrt {n\pi}}u^2\cos(u^2)du\\ &={4\over\pi n^{3/2}}\left( \left[ \dfrac{u\sin^2(u)}{2}\right]_0^{\sqrt{n\pi}} -\int_0^{\sqrt{n\pi}}\dfrac{\sin(u^2)}{2}du\right) \\ &= -{2\over \pi n^{3/2}} G(\sqrt{n\pi})\underset{n\to\infty}{\sim} -{2C\over \pi n^{3/2}}\quad\text{ (d'après la question précédente)}. \end{aligned}\]

  4. Vu ce qui précède, la série de Fourier de \(f\) : \(S_f(x) := {a_0\over 2}+\sum_{n\geq 1} a_n\cos(nx)\) converge normalement sur \(\mathbb R\), par suite \(S_f\in\mathscr C^0_{2\pi}(\mathbb R)\) ; la convergence normale nous dit aussi que \({a_0\over 2}+\sum_{n\geq 1} a_n\cos(nx)\) est la série de Fourier de \(S_f\) i.e. les deux fonctions continues \(f\) et \(S_f\) ont les mêmes coefficients de Fourier : donc \(f= S_f\) (par exemple via Parseval appliqué à \(f-S_f\)...) i.e. \[f(x) = {a_0\over 2}+\sum_{n\geq 1} a_n\cos(nx),\] \(f\) est donc bien développable en série de Fourier.

  5. Le théorème de Féjer1 assure que les sommes partielles de la série de Fourier d’une fonction \(f\) continue convergent au sens de Césaro vers \(f\), c’est donc ici le cas, mais la série de Fourier de \(f\) est aussi convergente sur \(\mathbb R\) et la convergence usuelle implique la convergence au sens de Césaro et vers la même limite : le résultat suit.


  1. 1  Voir par exemple....

Une fonction continue non dérivable à l’origine mais développable en série de Fourier (2) *

9 novembre 2022 22:30 — Par Patrice Lassère

Soit \(\displaystyle f(x)=\sum_{n\geq 1}\dfrac{\sin(nx)}{n^2}\).

  1. Montrer que \(f\in\mathscr C_0^{2\pi}\).

  2. Montrer que \(f\) est développable en série de Fourier sur \(\mathbb R\).

  3. On va montrer que \(f\) n’est pas dérivable à l’origine.

    \(\bullet\) Montrer que \(\sin(x)\geq \frac{2 x}{\pi},\ \forall\,x\in[0,\pi/2].\)

    \(\bullet\) Soit \(x\in]0,\pi/2[\), pour \(N={\rm{E}}(\pi/2x)\) (\(E\) est la partie entière...) montrer que \[\begin{aligned}\dfrac{f(x)}{x}&= \sum_{k=1}^N\dfrac{\sin(kx)}{kx}\dfrac{1}{k}+\dfrac{1}{x}\sum_{k\geq N+1}\dfrac{\sin(kx)}{k^2}\\ & \geq \dfrac{2}{\pi}\sum_{k=1}^N\dfrac{1}{k}+\dfrac{1}{x}\sum_{k\geq N+1}\dfrac{\sin(kx)}{k^2}:=\dfrac{2}{\pi}\sum_{k=1}^N\dfrac{1}{k}+R_N(x).\end{aligned}\]

    \(\bullet\) Montrer que \(\displaystyle R_N(x)=\sum_{k\geq N+1}\dfrac{\varphi_n(x)}{x}\left( \dfrac{1}{k^2}-\dfrac{1}{(k+1)^2}\right)\)\(\varphi_n(x)=\sum_{k=0}^n\sin(kx).\)

    \(\bullet\) Montrer que pour \(x\in\,]0,\pi/2[\) : \[\left\vert \varphi_n(x)\right\vert\leq \left\vert\dfrac{1-e^{i(n+1)x}}{1-e^{ix}}\right\vert\leq\dfrac{1}{\sin(x/2)}\leq \dfrac{\pi}{x}.\]

    \(\bullet\) En déduire que pour \(x\in\,]0,\pi/2[\) et \(N={\rm{E}}(\pi/2x)\) : \[\dfrac{f(x)}{x}\geq \dfrac{2}{\pi}\sum_{k=1}^N\dfrac{1}{k}-\dfrac{\pi}{N^2x^2}.\]

    \(\bullet\) En déduire que \(f\) n’est pas dérivable à l’origine.



[ID: 2911] [Date de publication: 9 novembre 2022 22:30] [Catégorie(s): Suites et séries de fonctions, séries entières ] [ Nombre commentaires: 0] [nombre d'éditeurs: 1 ] [Editeur(s): Emmanuel Vieillard-Baron ] [nombre d'auteurs: 1 ] [Auteur(s): Patrice Lassère ]
Accordéon
Titre
Solution
Texte

Une fonction continue non dérivable à l’origine mais développable en série de Fourier (2)
Par Patrice Lassère le 9 novembre 2022 22:30
  1. Comme \(\sup_{x\in\mathbb R}\vert \sin(nx)/n^2\vert=1/n^2\) la série converge normalement sur \(\mathbb R\) : \(f\) est donc continue sur \(\mathbb R\), elle bien entendu \(2\pi\)-périodique.

  2. La série définissant \(f\) est une série trigonométrique uniformément convergente sur \(\mathbb R\) : c’est donc la série de Fourier de \(f\).

    Pour justifier ce dernier point, si \(f(x)=\sum_{\mathbb Z}\alpha_k e^{ikx}\) où la convergence est uniforme sur un intervalle de longueur au moins \(2\pi\), il s’agit de montrer que pour tout \(k\in\mathbb Z\) : \(\alpha_k=c_k(f)\). Nous avons \[c_k(f)=\frac{1}{2\pi}\int_0^{2\pi}f(t)e^{-ikt}dt=\frac{1}{2\pi}\int_0^{2\pi}\sum_{l\in\mathbb Z}\alpha_l e^{ilt}e^{-ikt}dt =\frac{1}{2\pi}\sum_{l\in\mathbb Z}\alpha_l\int_0^{2\pi}e^{i(l-k)t}dt=\alpha_k\] où l’inversion \(\int\sum=\sum\int\) est justifiée par l’uniforme convergence de la série sur \([0,2\pi]\) ) la série de Fourier de \(f\).

  3. \(\bullet\) C’est une inégalité classique (inégalité de Jordan) souvent fort utile. Pour la démontrer on peut étudier la fonction ou mieux, invoquer la concavité de \(h\ :\ x\mapsto\sin(x)\) sur \([0,\pi/2]\) qui implique que \(h\) est au dessus de la corde reliant \((0,0)\) et \((\pi/2,1)\) i.e. la droite d’équation \(y=2x/\pi\).

    \(\bullet\) Soit \(x\in]0,\pi/2[\), pour \(N={\rm{E}}(\pi/2x)\) \[(\ 1\leq k\leq N\ ) \quad\implies\quad \left( 0< kx\leq {\rm{E}}(\pi/2x)x\leq \dfrac{\pi}{2}\right)\] on peut donc appliquer l’inégalité de Jordan \[\dfrac{\sin(kx)}{kx}\geq \dfrac{2 kx}{\pi kx}=\dfrac{2}{\pi},\quad\forall\,k\in\{1,\dots,N\},\] et par suite \[\begin{aligned}\dfrac{f(x)}{x}&= \sum_{k=1}^N\dfrac{\sin(kx)}{kx}\dfrac{1}{k}+\dfrac{1}{x}\sum_{k\geq N+1}\dfrac{\sin(kx)}{k^2}\\ & \geq \dfrac{2}{\pi}\sum_{k=1}^N\dfrac{1}{k}+\dfrac{1}{x}\sum_{k\geq N+1}\dfrac{\sin(kx)}{k^2}.\end{aligned}\]

    \(\bullet\) C’est une banale transformation d’Abel.

    \(\bullet\) Pour \(x\in\,]0,\pi/2[\) : \[\begin{aligned}\left\vert \varphi_n(x)\right\vert&=\left\vert\rm{Im}\left( \sum_{k=0}^n e^{ikx}\right) \right\vert \leq \left\vert\dfrac{1-e^{i(n+1)x}}{1-e^{ix}}\right\vert\\ &=\left\vert\dfrac{1-e^{i(n+1)x}}{e^{-ix/2}-e^{ix/2}}\right\vert \\ &\leq \dfrac{2}{\vert 2i\sin(x/2)\vert}\\ &\leq\dfrac{1}{\sin(x/2)}\leq \dfrac{\pi}{x}\end{aligned}\] où l’on a appliqué encore une fois l’inégalité de Jordan dans la dernière inégalité.

    \(\bullet\) D’où \[\begin{aligned} \dfrac{f(x)}{x}&\geq \dfrac{2}{\pi}\sum_{k=1}^N\dfrac{1}{k}+R_n(x)\\ &\geq \dfrac{2}{\pi}\sum_{k=1}^N\dfrac{1}{k}-\vert R_n(x)\vert\\ &\geq \dfrac{2}{\pi}\sum_{k=1}^N\dfrac{1}{k}-\dfrac{1}{x}\sum_{k\geq N+1}\vert\varphi_n(x)\vert\left( \dfrac{1}{k^2}-\dfrac{1}{(k+1)^2}\right) \\ &\geq \dfrac{2}{\pi}\sum_{k=1}^N\dfrac{1}{k}-\dfrac{1}{x} \sum_{k\geq N+1} \frac{\pi}{x}\left( \dfrac{1}{k^2}-\dfrac{1}{(k+1)^2}\right)\\ &=\dfrac{2}{\pi}\sum_{k=1}^N\dfrac{1}{k}-\dfrac{\pi}{N^2x^2}\\. \end{aligned}\]

    \(\bullet\) Si \(x\) tends vers zéro, \(N={\rm{E}}(\pi/2x)\) tends vers \(+\infty\) et \(N^2 x^2/\pi\) tends vers \(4/\pi\) de telle sorte que dans l’inégalité précédente le terme de droite tends vers l’infini avec \(N\) qui implique à son tour \(\lim_{x\to 0}\frac{f(x)}{x}=+\infty\) : \(f\) n’est pas dérivable à l’origine.


Une fonction continue dont la série de Fourier diverge à l’origine *

9 novembre 2022 22:30 — Par Patrice Lassère

On considère la suite de fonctions \((f_n)_{n\geq 1}\) définie sur \([0,\pi]\) par \[f_n(x)=\dfrac{1}{n^2}\sin\left[ \left( 2^{n^3}+1\right) \dfrac{x}{2}\right] .\]

  1. Montrer que la série de fonctions \(\sum_{n\geq 1}f_n\) converge normalement sur \([0,\pi]\).

    On désigne alors par \(f\) la fonction paire, continue, \(2\pi\)-périodique sur \(\mathbb R\) vérifiant pour tout \(x\in[0,\pi]\ :\quad f(x)=\sum_{n\geq 1}f_n(x)\).

  2. Montrer que \(f\in\mathscr C^0_{2\pi}\).

  3. On pose pour \(\displaystyle p,k\in\mathbb N,\quad I_{p,k}=\int_0^\pi\cos(pt)\sin\left( \dfrac{2k+1}{2}t\right) dt,\quad\) et pour tout entier \(q\in\mathbb N\quad:\quad T_{q,k}=\sum_{p=0}^q I_{p,k}\).

    1. Calculer, pour \(p,k\) entiers naturels l’intégrale \(I_{p,k}\).

    2. Pour \(q,k\in\mathbb N\), déterminer un réel positif \(c_k\) tel que \(\displaystyle T_{q,k}=c_k+\sum_{j=k-q}^{k+q}\dfrac{1}{2j+1}\).

    3. En déduire que \(\ T_{q,k}\geq 0\) pour tout couple \((q,k)\) d’entiers.

    4. Déterminer un équivalent simple de \(\ \sum_{k=0}^N\dfrac{1}{2k+1}\) au voisinage de \(+\infty\).

    5. En déduire que \(\displaystyle \ T_{k,k}\underset{k\to+\infty}{\sim}\dfrac{\log(k)}{2}.\)

  4. Montrer que pour tout \(\displaystyle p\in\mathbb N^\star,\quad a_p(f)=\dfrac{2}{\pi}\sum_{n\geq 1}\dfrac{1}{n^2}I_{p,2^{n^3-1}}.\)

  5. Montrer que pour tout \(\displaystyle p\in\mathbb N^\star,\quad S_{2^{p^3-1}}(f)(0)\geq -\dfrac{a_0(f)}{2}+\dfrac{2}{\pi p^2}T_{2^{p^3-1},2^{p^3-1}}\) (on pourra remarquer que \(\frac{a_0(f)}{2}+\sum_{l=1}^Na_l(f)=-\frac{a_0(f)}{2}+\sum_{l=0}^Na_l(f)\)...).

  6. En déduire que la suite \((S_n(f)(0))_n\) diverge.



[ID: 2913] [Date de publication: 9 novembre 2022 22:30] [Catégorie(s): Suites et séries de fonctions, séries entières ] [ Nombre commentaires: 0] [nombre d'éditeurs: 1 ] [Editeur(s): Emmanuel Vieillard-Baron ] [nombre d'auteurs: 1 ] [Auteur(s): Patrice Lassère ]
Accordéon
Titre
Solution
Texte

Une fonction continue dont la série de Fourier diverge à l’origine
Par Patrice Lassère le 9 novembre 2022 22:30
  1. La convergence normale est évidente (\(\Vert f_n\Vert_\infty= 1/n^2\) ) la série \(\sum_{n\geq 1}f_n\) est donc une fonction continue sur \([0,\pi]\).

  2. Par parité et prolongement \(2\pi\)-périodique, \(\lim_{x\to -\pi_+}f(x)=\lim_{x\to \pi_+}f(x)=\lim_{x\to \pi_-}f(x)\) : \(f\) est donc bien continue en les points \(k\pi,\ k\in\mathbb Z\) ce qui était le seul point douteux.

  3. a) De la formule \(\sin(a)\cos(b)=\frac{1}{2}(\sin(a+b)+\sin(a-b))\) on tire \[\begin{aligned}I_{p,k}&=\int_0^\pi\cos(pt)\sin\left( \dfrac{2k+1}{2}t\right) dt \\ &=\dfrac{1}{2}\int_0^\pi \left[ \sin\left( \left( \dfrac{2k+1}{2}+p\right) t\right)+\sin\left( \left( \dfrac{2k+1}{2}-p\right) t\right) \right] dt\\&= -\dfrac{1}{2}\left[ \dfrac{ \cos\left( \left( \dfrac{2k+1}{2}+p\right) t\right)}{ \dfrac{2k+1}{2}+p}+\dfrac{ \cos\left( \left( \dfrac{2k+1}{2}+p\right) t\right)}{ \dfrac{2k+1}{2}-p}\right]_0^\pi\\ &=\dfrac{1}{2(k+p)+1}+\dfrac{1}{2(k-p)+1} \end{aligned}\]

    b) On peut alors écrire : \[\begin{aligned} T_{q,k}&=\sum_{p=0}^q I_{p,k}=\sum_{p=0}^q\dfrac{1}{2(k+p)+1}+\sum_{p=0}^q\dfrac{1}{2(k-p)+1}\\ &=\sum_{j=k}^{k+q}\dfrac{1}{2j+1}+\sum_{j=k-q}^{k}\dfrac{1}{2j+1}\\ &=\dfrac{1}{2k+1}+\sum_{j=k-q}^{k+q}\dfrac{1}{2j+1}\\ &=c_k+\sum_{j=k-q}^{k+q}\dfrac{1}{2j+1} \end{aligned}\] avec \(c_k=\dfrac{1}{2k+1}>0\).

    c) Si \(k\geq q\) il est évident que \(T_{q,k}\geq 0\).

    Maintenant, si \(k<q\) il suffit de remarquer que \[\sum_{j=k-q}^{k+q}\dfrac{1}{2j+1}=\sum_{j=k-q}^{q-k}\dfrac{1}{2j+1}+\sum_{j=q-k+1}^{k+q}\dfrac{1}{2j+1}=\dfrac{1}{2(q-k)+1}+\sum_{j=q-k+1}^{k+q}\dfrac{1}{2j+1}\geq 0\] puisque \(q-k+1\geq 0.\)

    d) Par décroissance de \(x\mapsto (2x+1)^{-1}\) sur \(\mathbb R_+\) on peut écrire pour tout \(N\in\mathbb N^\star\) \[\int_1^N\dfrac{dt}{2t+1}\leq \sum_{k=0}^N\dfrac{1}{2k+1}\leq \int_0^N\dfrac{dt}{2t+1},\] soit \[\dfrac{\log(2N+1)-\log(3)}{2}\leq \sum_{k=0}^N\dfrac{1}{2k+1}\leq \dfrac{\log(2N+1)}{2}\] qui implique \(\quad\displaystyle \sum_{k=0}^N\dfrac{1}{2k+1}\underset{N\to\infty}{\sim}\dfrac{\log(N)}{2}.\)

    e) Avec (15-b) et (15-d) \[T_{k,k}=\dfrac{1}{2k+1}+\sum_{j=0}^{2k}\dfrac{1}{2j+1}\underset{k\to\infty}{\sim}\dfrac{\log(2k)}{2}\underset{k\to\infty}{\sim}\dfrac{\log(k)}{2}.\]

  4. Soit \(p\in\mathbb N^\star\) \[\begin{aligned}a_p(f)&=\dfrac{1}{\pi}\int_{-\pi}^\pi f(t)\cos(pt)dt=\dfrac{2}{\pi}\int_{0}^\pi \sum_{n\geq 1}f_n(t)\cos(pt)dt\\ &=\dfrac{2}{\pi}\sum_{n\geq 1}\int_{0}^\pi f_n(t)\cos(pt)dt\qquad(\text{par NCV (question 13) sur }\ [0,\pi])\\ &=\dfrac{2}{\pi}\sum_{n\geq 1}\int_{0}^\pi \dfrac{1}{n^2}\sin\left[ \left( 2^{n^3}+1\right) \dfrac{t}{2}\right]\cos(pt)dt\\ &=\dfrac{2}{\pi}\sum_{n\geq 1}\dfrac{1}{n^2}I_{p,2^{n^3-1}}. \end{aligned}\]

  5. De l’égalité précédente et sachant que (15-c) les quantités \(T_{p,k}\) sont toujours positives, on a pour tout \(p\geq 1\) \[a_p(f)\geq \dfrac{2}{\pi n^2}I_{p,2^{n^3-1}},\quad\forall\,n\in\mathbb N^\star.\] Ainsi \[\begin{aligned} S_{2^{N^3-1}}(f)(0)&=\dfrac{a_0(f)}{2}+\sum_{n=1}^{2^{N^3-1}} a_n(f)=-\dfrac{a_0(f)}{2}+\sum_{n=0}^{2^{N^3-1}} a_n(f)\\ &\geq -\dfrac{a_0(f)}{2}+\sum_{n=0}^{2^{N^3-1}}\dfrac{2}{\pi {N}^2}I_{n,2^{N^3-1}}\\ &=-\dfrac{a_0(f)}{2}+\dfrac{2}{\pi {N}^2}T_{2^{N^3-1},2^{N^3-1}} \end{aligned}\]

  6. L’équivalent obtenu en (15-e) nous donne \[\dfrac{2}{\pi {N}^2}T_{2^{N^3-1},2^{N^3-1}}\underset{N\to\infty}{\sim} \dfrac{2}{\pi {N}^2} \dfrac{\log(2^{N^3-1})}{2}\underset{N\to\infty}{\sim}\dfrac{N^3\log(2)}{\pi N^2}=\dfrac{N\log(2)}{\pi}.\] Cette dernière quantité tendant vers \(+\infty\) avec \(N\), on a avec la question (17) \[\lim_{N\to\infty} S_{2^{N^3-1}}(f)(0)=+\infty.\] La suite \((S_n(f)(0))_n\) est donc divergente : \(f\) n’est pas développable en série de Fourier à l’origine.


Séries de Fourier, dérivation *

9 novembre 2022 22:31 — Par Patrice Lassère

[rms], 2003.

Soit \(f\in\mathscr C^2([0,1],\mathbb R)\) vérifiant \(f(0)=f(1)=0\). Avec les séries de Fourier, montrer que \[\Vert f\Vert_ \infty\leq \dfrac{\Vert f''\Vert_2}{3\sqrt 5}.\]



[ID: 2915] [Date de publication: 9 novembre 2022 22:31] [Catégorie(s): Suites et séries de fonctions, séries entières ] [ Nombre commentaires: 0] [nombre d'éditeurs: 1 ] [Editeur(s): Emmanuel Vieillard-Baron ] [nombre d'auteurs: 1 ] [Auteur(s): Patrice Lassère ]
Accordéon
Titre
Solution
Texte

Séries de Fourier, dérivation
Par Patrice Lassère le 9 novembre 2022 22:31

La fonction \(f\) étant de classe \(\mathscr C^2\) et vérifiant \(f(0)=f(1)=0\) il existe une unique fonction \(\tilde f\) impaire, \(2\)-périodique dont la restriction à \([0,1]\) est \(f\). On vérifie sans peine que cette fonction est \(\mathscr C^1\) sur \(\mathbb R\) (et même \(\mathscr C^2\) par morceaux). Calculons ses coefficients de Fourier (complexes), bien entendu \(c_0(\tilde f)=0\) et pour \(n\in\mathbb Z^\star\) une intégration par parties donne \[c_n(\tilde f)=\dfrac{1}{2}\int_{-1}^1\tilde f(t)e^{-int}dt=\dfrac{1}{2i\pi n}\int_{-1}^1\tilde f'(t)e^{-int}dt\] une seconde intégration par parties est licite puisque \(\tilde f'\) est continue et \(\mathscr C^1\) par morceaux : \[c_n(\tilde f)=\dfrac{1}{2i\pi n}\left[ \dfrac{\tilde f'(t)e^{-int}}{-i\pi n}\right]_{-1}^1 + \dfrac{1}{2(i\pi n)^2}\int_{-1}^1\tilde f''(t)e^{-int}dt=-\dfrac{c_n(\tilde f'')}{\pi^2n^2}\] (le terme entre crochets est nul car \(\tilde f'\) est paire). Vu la régularité de \(\tilde f\), les théorèmes classiques nous assurent que la série de Fourier de \(\tilde f\) est normalement convergente sur \(\mathbb R\) avec pour somme \(\tilde f\), soit \[\tilde f(x)=\sum_{n\in\mathbb Z}c_n(f)e^{in\pi x}=-\dfrac{1}{\pi^2}\sum_{n\in\mathbb Z^\star}\dfrac{c_n(\tilde f'')}{n^2}e^{in\pi x},\quad\forall\,x\in\mathbb R.\] En particulier \[\forall\,x\in[0,1]\quad :\quad \vert f(x)\vert=\vert\tilde f(x)\vert\leq\dfrac{1}{\pi^2}\sum_{n\in\mathbb Z^\star}\dfrac{\vert c_n(\tilde f'')}{n^2}\] soit en appliquant l’inégalité de Cauchy-Schwarz (\(\sum_{n\geq 1}n^{-4}=\frac{\pi^4}{90}\)) \[\begin{aligned}\forall\,x\in[0,1]\quad :\quad \vert f(x)\vert^2&\leq \dfrac{1}{\pi^4}\sum_{n\in\mathbb Z^\star}\vert c_n(\tilde f'')\vert^2\sum_{n\in\mathbb Z^\star}\dfrac{1}{n^4}\\ &\leq\dfrac{2}{\pi^4}\sum_{n\in\mathbb Z^\star}\vert c_n(\tilde f'')\vert^2\sum_{n\geq 1}\dfrac{1}{n^4}=\dfrac{1}{45}\sum_{n\in\mathbb Z^\star}\vert c_n(\tilde f'')\vert^2 \end{aligned}\] \(\tilde f''\) étant \(2\)-périodique et continue par morceaux, on peut lui appliquer le théorème de Parseval : \[\dfrac{1}{2}\int_{-1}^1\vert \tilde f''(t)\vert^2dt=\sum_{n\in\mathbb Z^\star}\vert c_n(\tilde f'')\vert^2,\] si bien que pour tout \(x\in[0,1]\) \[\vert f(x)\vert \leq \dfrac{1}{45}\Vert f''\Vert_2^2\] soit \[\Vert f\Vert_ \infty\leq \dfrac{\Vert f''\Vert_2}{3\sqrt 5}\] d’où le résultat.


Séries entières, déterminant, systèmes linéaires *

9 novembre 2022 22:31 — Par Patrice Lassère

Soit \(a,b,c,d\) des nombres complexes, si \(d\ne 0\) et soit la suite \((c_n)_n\subset\mathbb C\) telle que \[\dfrac{az+b}{z^2+cz+d}=c_0+c_1z+c_2z^2+\dots+c_nz^n\dots\] pour \(\vert z\vert\) assez petit. Montrer que la quantité \[\dfrac{\text{det}\begin{pmatrix}c_n &c_{n+1}\\c_{n+1}&c_{n+2}\end{pmatrix}} {\text{det}\begin{pmatrix}c_{n+1} &c_{n+2}\\c_{n+2}&c_{n+3}\end{pmatrix}}\] est indépendante de \(n\) lorsque \(abc-b^2-ad^2\ne 0\).



[ID: 2917] [Date de publication: 9 novembre 2022 22:31] [Catégorie(s): Suites et séries de fonctions, séries entières ] [ Nombre commentaires: 0] [nombre d'éditeurs: 1 ] [Editeur(s): Emmanuel Vieillard-Baron ] [nombre d'auteurs: 1 ] [Auteur(s): Patrice Lassère ]
Accordéon
Titre
Solution
Texte

Séries entières, déterminant, systèmes linéaires
Par Patrice Lassère le 9 novembre 2022 22:31
  1. Solution 1 : De l’égalité \[az+b=(z^2+cz+d)\sum_{n\geq 0}c_nz^n\] on tire \[c_0=\dfrac{b}{d},\quad c_1=\dfrac{ad-bc}{d^2}\quad\text{et}\quad dc_{n+2}+cc_{n+1}+c_n=0,\ n\in\mathbb N\] si bien que \[\begin{aligned}c_{n+1}c_{n+3}-c_{n+2}^2&=c_{n+1}\left( -\dfrac{c}{d}c_{n+2}-\dfrac{1}{d}c_{n+1}\right)-c_{n+2}^2 \\ &=-\dfrac{c}{d}c_{n+1}c_{n+2}-c_{n+2}^2-\dfrac{1}{d}c_{n+1}^2\\ &=c_{n+2}\left( -\dfrac{c}{d}c_{n+1}-c_{n+2}\right) -\dfrac{1}{d}c_{n+1}^2\\ &=\dfrac{1}{d}\left( c_{n+2}c_{n}-c_{n+1}^2\right) \\ &\dots\\ &=\dfrac{1}{d^{n+1}}\left( c_{2}c_{0}-c_{1}^2\right). \end{aligned}\] -Ainsi pour \(c_{2}c_{0}-c_{1}^2\ne 0\ \) i.e. \(\ abc-b^2-a^2d\ne 0\) : \[\dfrac{\text{det}\begin{pmatrix}c_n &c_{n+1}\\c_{n+1}&c_{n+2}\end{pmatrix}} {\text{det}\begin{pmatrix}c_{n+1} &c_{n+2}\\c_{n+2}&c_{n+3}\end{pmatrix}}=d\] est bien indépendant de \(n\).

    -Et pour \(abc-b^2-a^2d= 0\) on a \[\text{det}\begin{pmatrix}c_n &c_{n+1}\\c_{n+1}&c_{n+2}\end{pmatrix}= \text{det}\begin{pmatrix}c_{n+1} &c_{n+2}\\c_{n+2}&c_{n+3}\end{pmatrix}=0\] et le quotient n’est pas défini.

  2. Solution 2 : Toujours de l’égalité \[az+b=(z^2+cz+d)\sum_{n\geq 0}c_nz^n\] comparant les coefficients de \(z^{n+2}\) et \(z^{n+3}\) des deux cotés on tire \[\begin{cases}c_n+cc_{n+1}+dc_{n+2}&= 0\\ c_{n+1}+cc_{n+2}+dc_{n+3}&=0\end{cases}\] En considérant ces deux égalités comme un système linéaire d’inconnues \(c\) et \(d\), les régles de Kramer donnent \[d= \dfrac{\text{det}\begin{pmatrix}c_n &c_{n+1}\\c_{n+1}&c_{n+2}\end{pmatrix}} {\text{det}\begin{pmatrix}c_{n+1} &c_{n+2}\\c_{n+2}&c_{n+3}\end{pmatrix}}\] pourvu que le dénominateur ne s’annule pas, i.e. \[\text{det}\begin{pmatrix}c_{n+1} &c_{n+2}\\c_{n+2}&c_{n+3}\end{pmatrix}=abc-b^2-a^2d\ne 0.\]


Étude de \(f(x) = \sum^{ \infty}_{ n=1}\sin (nx) \exp \left(-n^a \right)\) *

9 novembre 2022 22:31 — Par Patrice Lassère

[rms], 2003/04.

Pour \(a > 0\), \(x\) réel quelconque, on pose \(f(x) = \sum^{ \infty}_{ n=1}\sin (nx) \exp \left(-n^a \right)\).

  1. Montrer que \(f\) est bien définie et de classe \({\mathscr C}^{\infty}\).

  2. Pour quelles valeurs de \(a\), \(f\) est-elle développable en série entière au voisinage de tout point ?



[ID: 2919] [Date de publication: 9 novembre 2022 22:31] [Catégorie(s): Suites et séries de fonctions, séries entières ] [ Nombre commentaires: 0] [nombre d'éditeurs: 1 ] [Editeur(s): Emmanuel Vieillard-Baron ] [nombre d'auteurs: 1 ] [Auteur(s): Patrice Lassère ]
Accordéon
Titre
Solution
Texte

Étude de \(f(x) = \sum^{ \infty}_{ n=1}\sin (nx) \exp \left(-n^a \right)\)
Par Patrice Lassère le 9 novembre 2022 22:31
  1. Posons \(u_n(x)=e^{inx}e^{-n ^a}\). La fonction \(u_n\) est de classe \({\mathscr C}^\infty\) sur \(\mathbb{R}\), et si \((n,k)\in\mathbb{N}^*\times \mathbb{N}\) et \(x\in\mathbb{R}\) :

    \[{u_n}^{(k)}(x)=(in)^ke^{inx}e^{-n ^a}.\]

    Notant \(\vert\vert\;\vert\vert _\infty\) la norme uniforme sur \(\mathbb{R}\), on a donc \(\vert\vert{u_n}^{(k)}\vert\vert _\infty =n^ke^{-n ^a}\). Par croissance comparée il vient, à \(k\) fixé : \(\vert\vert{u_n}^{(k)}\vert\vert _\infty =o(1/n^2)\) quand \(n\rightarrow +\infty\) de sorte que \(\sum_{n\geqslant 1}{u_n}^{(k)}\) coverge normalement donc uniformément sur \(\mathbb{R}\). Ceci étant vrai pour tout \(k\), \(\sum_{n\geqslant 1} u_n\) est de classe \({\mathscr C}^\infty\) sur \(\mathbb{R}\), et il en va de même de sa partie imaginaire \(f\).

  2. Soit \(g(x)=\sum_{n\geqslant 1}e^{inx}e^{-n ^a}\) de sorte que \(f\)=Im\(\;(g)\). Si \((x,h)\in\mathbb{R}^2\) est tel que la série double :

    \[\sum_{n\geqslant 1,\; k\geqslant 1}e^{-n ^a}\frac{(n\vert h\vert)^k}{k!}\]

    converge, le calcul suivant est justifié :

    \[g(x+h)=\sum_{n\geqslant 1}e^{-n ^a}e^{inx}\left( \sum_{k\geq O}\frac{(inh)^k)}{k!} \right)=\sum_{k\geq 0}\left(\sum_{n\geq 1}(in)^ke^{-n^\alpha}e^{inx}\right) \frac{h^k}{k!} =\sum_{k\geqslant 0}\frac{g^{(k)}(x)}{k!}h^k.\]

    Or, si \(a>1\), la relation :

    \[\sum_{k\geqslant 0}\frac{(n\vert h\vert)^k}{k!}=e^{n\vert h\vert},\]

    la positivité des \(e^{-n ^a}(n\vert h\vert)^k/k!\) et la convergence de \(\sum_{n\geqslant 1}e^{n\vert h\vert}e^{-n ^a}\) (immédiate par exemple par la règle de d’Alembert) établissent bien la convergence de la série double susmentionnée. Il s’ensuit que :

    \[\forall (x,h)\in\mathbb{R}^2,\qquad g(x+h) =\sum_{k\geqslant 0}\frac{g^{(k)}(x)}{k!}h^k.\]

    Prenant les parties imaginaires, on obtient :

    \[\forall (x,h)\in\mathbb{R}^2, \qquad f(x+h)=\sum_{k\geqslant 0}\frac{f^{(k)}(x)}{k!}h^k.\]

    Si \(a=1\), la série double précédente converge pourvu que \(\vert h\vert<1\) (toujours par la règle de d’Alembert). Ainsi :

    \[\forall (x,h)\in\mathbb{R}\times ]-1,1[, \qquad f(x+h)=\sum_{k\geqslant 0}\frac{f^{(k)}(x)}{k!}h^k.\]

    Dans ce cas, on peut par ailleurs observer que :

    \[g(x)=\frac{e^{(ix-1)}}{1-e^{(ix-1)}}.\]

    Montrons enfin, si \(0<a<1\), que la série :

    \[\sum_{k\geqslant 0}\frac{f^{(k)}(0)}{k!}h^k\]

    ne converge pour aucune valeur de \(h\) dans \(\mathbb{R}^*\). Les arguments donnés dans a) prouvent que :

    \[\forall k\in\mathbb{N},\quad \forall x\in\mathbb R,\quad f^{(4k+1)}(x)= \sum_{n\geqslant 0}n^{4k+1}e^{-n ^a}\cos (nx).\]

    En particulier :

    \[f^{(4k+1)}(0)=\sum_{n\geqslant 0}n^{4k+1}e^{-n ^a}.\]

    Soit, si \(k\in\mathbb{N}\) : \(n_k=E\left( (4k+1)^{1/a}\right)\). Alors :

    \[f^{(4k+1)}(0)\geqslant {n_k}^{4k+1}e^{-{n_k} ^a}\geqslant \left( (4k+1)^{1/a}-1\right)^{4k+1} e^{-(4k+1)}.\]

    D’où, vu que \(1/a>1\) :

    \[\frac{f^{(4k+1)}(0)}{(4k+1)!}\geqslant \frac{(4k)^{(4k+1)/a}e^{-(4k+1)}}{(4k+1)!}.\]

    Si :

    \[u_k=\frac{(4k)^{(4k+1)/a}e^{-(4k+1)}}{(4k+1)!},\]

    \(u_{k+1}/u_k\) tend vers \(+\infty\) par un calcul immédiat. Il en résulte aussitôt que :

    \[\sum_{k\geqslant 0}\frac{f^{(4k+1)}(0)}{(4k+1)!}h^{4k+1}\]

    ne converge que pour \(h=0\) puis, compte tenu de la convergence absolue d’une série entière en tout point du disque ouvert de convergence, que :

    \[\sum_{k\geqslant 0}\frac{f^{(k)}(0)}{k!}h^k\]

    ne converge que pour \(h=0\).

    En conclusion, \(f\) est développable en série entière au voisinage de tout point si et seulement si \(a\in]0,1]\).

    Remarque : Notons \({\mathscr E}\) l’ensemble des suites \((c_n)_{n\in\mathbb{Z}}\) telles qu’il existe \(r\) dans \(]0,1[\) vérifiant \(c_n=O(r^{\vert n\vert})\) quand \(n\rightarrow \pm\infty\). On peut démontrer que l’application qui à une fonction continue sur \(\mathbb{R}\) et \(2\pi\)-périodique associe la suite \((c_n(f))_{n\in\mathbb{Z}}\) de ses coefficients de Fourier exponentiels induit une bijection de l’espace des fonctions \(2\pi\)-périodiques analytiques de \(\mathbb{R}\) dans \(\mathbb{C}\) sur \({\mathscr E}\), la bijection réciproque associant à \((c_n)_{n\in\mathbb{Z}}\) la fonction :

    \[x\mapsto \sum_{-\infty}^{+\infty}c_ne^{inx}.\]

    Avec cette caractérisation, le résultat de l’exercice est immédiat.


Séries entières, comportement au bord *

9 novembre 2022 22:31 — Par Patrice Lassère

Soit \(\sum_{n\geq 0} a_nx^n\) une série entière de rayon de convergence égal à \(1\). On suppose que \(\forall\,n\geq 1\ :\ a_n\geq 0\). Si la série \(\sum_n a_n\) diverge, montrer que \(\displaystyle\lim_{x\to 1_-}\sum_n a_nx^n=+\infty\).



[ID: 2921] [Date de publication: 9 novembre 2022 22:31] [Catégorie(s): Suites et séries de fonctions, séries entières ] [ Nombre commentaires: 0] [nombre d'éditeurs: 1 ] [Editeur(s): Emmanuel Vieillard-Baron ] [nombre d'auteurs: 1 ] [Auteur(s): Patrice Lassère ]
Accordéon
Titre
Solution
Texte

Séries entières, comportement au bord
Par Patrice Lassère le 9 novembre 2022 22:31

Vu les hypothèses, \(\forall\,A>0,\ \exists\, m\in\mathbb N\ :\ a_0+a_1+\dots+a_m>A+1\) et par continuité de \(x\mapsto a_0+a_1x+\dots+a_mx^m\) il existe \(0<\delta_{A}<1\) tel que pour tout \(x\in\,]1-\delta_A,1[\ :\ a_0+a_1x+\dots+a_mx^m>A\). Tout étant positif, à fortiori \(\sum_n\,a_nx^n\geq a_0+a_1x+\dots+a_mx^m>A\). En résumé

\[\left(\forall\,A>0,\ \exists\,\delta_A\in]0,1[\ :\ \forall\,x\in]1-\delta_A,1[,\ \sum_n\,a_nx^n>A\right)\Longrightarrow \lim_{x\to 1_-}\sum_n a_nx^n=+\infty.\]


Séries de Fourier : histoires d’unicité *

9 novembre 2022 22:31 — Par Patrice Lassère

Montrer que pour tout \(a\neq 0\) \[e^{ax}=\begin{cases} \dfrac{e^{2a\pi}-1}{\pi}\left( \dfrac{1}{2a}+\displaystyle\sum_{n=1}^\infty\dfrac{a\cos(nx)-n\sin(nx)}{a^2+n^2}\right),\quad 0<x<2\pi\\ \dfrac{e^{a\pi}-1}{a\pi}+\dfrac{2}{\pi}\displaystyle\sum_{n=1}^\infty\left\lbrace (-1)^ne^{a\pi}-1\right\rbrace \dfrac{a\cos(nx)}{a^2+n^2},\quad 0<x<\pi\\ \dfrac{2}{\pi}\displaystyle\sum_{n=1}^\infty\left\lbrace 1-(-1)^ne^{a\pi}\right\rbrace \dfrac{n\sin(nx)}{a^2+n^2},\quad 0<x<\pi. \end{cases}\] et préciser les sommes de ces trois séries à l’origine.



[ID: 2923] [Date de publication: 9 novembre 2022 22:31] [Catégorie(s): Suites et séries de fonctions, séries entières ] [ Nombre commentaires: 0] [nombre d'éditeurs: 1 ] [Editeur(s): Emmanuel Vieillard-Baron ] [nombre d'auteurs: 1 ] [Auteur(s): Patrice Lassère ]
Accordéon
Titre
Solution
Texte

Séries de Fourier : histoires d’unicité
Par Patrice Lassère le 9 novembre 2022 22:31

La fonction \(f\) définie par \(f(x)=e^{ax}\) pour tout \(x\in\,]0,2\pi[\) et \(f(0)=\frac{1+e^{2a\pi}}{2}\), prolongée sur \(\mathbb R\) par \(2\pi\)-périodicité vérifie les hypothèses du théorème de Jordan-Dirichlet : la série de Fourier de \(f\) converge sur \(\mathbb R\) vers \(f\) ; ce qui donne après un petit calcul la première formule. Pour la seconde il suffit de considèrer la fonction \(g\) \(2\pi\)-périodique, paire et égale à \(f\) sur \([0,\pi]\). Pour la dernière, considèrer la fonction \(h\) \(2\pi\)-périodique, impaire et égale à \(f\) sur \(]0,\pi[\) et vérifiant \(h(0)=h(\pi)=h(-\pi)=0\). Jordan-Dirichlet s’appliquant au trois fonctions, les trois séries convergent à l’origine respectivement vers \(f(0),g(0),h(0).\)

Remarque : Moralité, les problème d’unicité pour les séries trigonométriques sont inifiement plus subtils que pour les séries entières ; ce n’est pas parce qu’une série trigonométrique converge vers une fonction qu’elle est la série de Fourier de cette fonction, surtout si le domaine de convergence est un intervalle de longueur strictement plus petite que \(2\pi\). Une condition suffisante pour pouvoir affirmer une telle chose que l’on recontre souvent est la convergence uniforme sur un intervalle de longueur supérieure ou égale à \(2\pi\). Toutefois, si la série de Fourier d’une fonction continue converge en un point, ce sera nécessairement vers la valeur de la fonction en ce point. Pour s’en persuader, il faut se souvenir que la convergence usuelle implique la convergence au sens de Césaro vers la même limite et que la série de Fourier d’une fonction continue converge toujours simplement au sens de Césaro vers la fonction1.


  1. 1  voir l’exercice ??

SON et SCV *

9 novembre 2022 22:31 — Par Patrice Lassère

0n considère une suite \((f_n)_n\subset\mathscr C^0([0,1])\) vérifiant \[\int_0^1f_n(t)f_m(t)dt=\begin{cases} 0\quad \rm{si}\quad &n=m\\ 1\quad \rm{sinon}\quad &n\neq m\end{cases}\] et \[\sup\{ \vert f_n(x)\vert\ :\ x\in\mathbb R,\ n\in\mathbb N\,\}<+\infty.\] Montrer que la suite \((f_n)_n\) n’admet pas de sous-suite \((f_{n_k})_k\) simplement convergente sur \([0,1]\).



[ID: 2925] [Date de publication: 9 novembre 2022 22:31] [Catégorie(s): Suites et séries de fonctions, séries entières ] [ Nombre commentaires: 0] [nombre d'éditeurs: 1 ] [Editeur(s): Emmanuel Vieillard-Baron ] [nombre d'auteurs: 1 ] [Auteur(s): Patrice Lassère ]
Accordéon
Titre
Solution
Texte

SON et SCV
Par Patrice Lassère le 9 novembre 2022 22:31

-On commence par remarquer qu’on ne pert pas en généralité en supposant que \(\overline{\rm{vect}\{f_n\}}_{L^2}=L^2([0,1])\) : en effet, si ce n’est pas le cas on rajoute à la suite \((f_n)_n\) une suite \((g_n)_n\) vérifiant les mêmes hypothèses mais aussi \(\overline{\rm{vect}\{f_n\}\cup\{g_m\}}_{L^2}=L^2([0,1])\) (une telle suite \((g_n)\) existe : considérer pour cela la base hilbertienne .......).

-Supposons maintenant qu’il existe une sous-suite \((f_{n_k})_k\) et une application \(f\in\mathscr C^0([0,1])\) telles que \[\lim_{k\to\infty}f_{n_k}(x)=f(x),\qquad \forall\,x\in [0,1].\] Soit \(m\in\mathbb N\), la seconde hypothèse sur la suite \((f_n)_n\) nous permet d’appliquer le théorème de la convergence dominée : \[0=\lim_{k\to\infty}\int_0^1 f_{n_k}(t)f_m(t)dt=\int_0^1 f(t)f_m(t)dt,\] soit \[\forall\,m\in\mathbb N\quad :\quad \int_0^1 f(t)f_m(t)dt=0\] et par conséquent \(f=0_{L^2}\) est donc nulle presque partout. Mais toujours par convergence dominée \[1=\lim_{k\to\infty}\int_0^1 f_{n_k}^2 (t)dt=\int_0^1f^2(t)dt,\] tout ceci est absurde et l’exercice est terminé.


Fonction \(2\pi\)-périodique continue à coefficients de Fourier positifs *

9 novembre 2022 22:31 — Par Patrice Lassère

[rms], 115/4.

Soit \(f\ :\ \mathbb R\to\mathbb C\) une application \(2\pi\) périodique et continue. On suppose tous ses coefficients de Fourier (\(c_k(f),\ k\in\mathbb Z\)) positifs ; montrer que \(f\) est développable en série de Fourier.

Pour cela

  1. Montrer que pour tout \(0<r<1\) \[\sum_{n\in\mathbb Z} c_n(f)r^{\vert n\vert}=\dfrac{1}{2\pi}\int_0^{2\pi}\dfrac{1-r^2}{1-2r\cos(t)+r^2}f(t)dt:=\dfrac{1}{2\pi}\int_0^{2\pi}P_r(t)f(t)dt\]

  2. En déduire que \[\dfrac{1}{2\pi}\int_0^{2\pi}P_r(t)dt=1\quad\text{et}\quad\sum_{n\in\mathbb Z} c_n(f)r^{\vert n\vert}\leq \Vert f\Vert_\infty.\]

  3. En déduire que la série \(\sum_{n\in\mathbb Z}c_n(f)\) converge et conclure.



[ID: 2927] [Date de publication: 9 novembre 2022 22:31] [Catégorie(s): Suites et séries de fonctions, séries entières ] [ Nombre commentaires: 0] [nombre d'éditeurs: 1 ] [Editeur(s): Emmanuel Vieillard-Baron ] [nombre d'auteurs: 1 ] [Auteur(s): Patrice Lassère ]
Accordéon
Titre
Solution
Texte

Fonction \(2\pi\)-périodique continue à coefficients de Fourier positifs
Par Patrice Lassère le 9 novembre 2022 22:31
  1. Soit \(0<r<1\) \[\begin{aligned} \sum_{n\in\mathbb Z} c_n(f)r^{\vert n\vert} &= \sum_{n\in\mathbb Z} \dfrac{1}{2\pi}\int_0^{2\pi} f(t)e^{-int}r^{\vert n\vert}dt\\ &=\dfrac{1}{2\pi}\int_0^{2\pi}\sum_{n\in\mathbb Z}f(t)e^{-int}r^{\vert n\vert}dt\qquad\text{par NCV sur}\ [0,2\pi]\\ &=\dfrac{1}{2\pi}\int_0^{2\pi}f(t)\left( \sum_{n=0}^\infty e^{-int}r^{n}+\sum_{n=1}^{\infty} e^{int}r^{n}\right) dt\\ &=\dfrac{1}{2\pi}\int_0^{2\pi}f(t)\left(\dfrac{1}{1-re^{-it}}+\dfrac{re^{it}}{1-re^{it}}\right)dt \\ &=\dfrac{1}{2\pi}\int_0^{2\pi}f(t)\dfrac{1-r^2}{1-2r\cos(t)+r^2}dt \end{aligned}\]

  2. En considérant \(f\equiv 1\) la formule précédente se resume à \[\dfrac{1}{2\pi}\int_0^{2\pi}f(t)P_r(t)dt=\dfrac{1}{2\pi}\int_0^{2\pi}P_r(t)dt=c_0(f)=1.\] De là, pour \(f\in\mathscr C_{2\pi}^0\) \[\sum_{n\in\mathbb Z} c_n(f)r^{\vert n\vert}=\dfrac{1}{2\pi}\int_0^{2\pi}P_r(t)f(t)dt \leq \dfrac{1}{2\pi}\int_0^{2\pi}\vert f(t)\vert P_r(t)dt\leq \Vert f\Vert_\infty \dfrac{1}{2\pi}\int_0^{2\pi} P_r(t)dt=\Vert f\Vert_\infty.\] Soit \(N\in\mathbb N\) nous pouvons donc écrire \[0\leq \sum_{k=-N}^N c_k(f)r^{\vert n\vert}\leq \Vert f\Vert_\infty,\qquad\forall\,r\in]0,1[,\] soit, si \(r\) tends vers \(1_-\) \[0\leq \sum_{k=-N}^N c_k(f)\leq \Vert f\Vert_\infty,\ \forall\,n\in\mathbb N.\]

  3. Les coefficients de Fourier de \(f\) étant positifs, la convergence de la série \(\sum_{k\in\mathbb Z}c_k(f)\) en découle ; elle entraine la normale convergence sur \(\mathbb R\) de la série de Fourier de \(f\).

    Il ne reste plus qu’à se souvenir que si la série de Fourier d’une application \(f\in\mathscr C^0_{2\pi}\) converge uniformément sur un intervale de longueur supérieure à \(2\pi\) c’est nécessairement vers \(f\) ( poser \(g=\sum_\mathbb Z c_k(f)e^{ikx}\), montrer que \(c_k(f)=c_k(g),\ \forall\,k\), et en déduire via Parseval de \(f-g\equiv 0\)).


\(\int_0^1( \int_0^1 f(x,y)dx)^2 dy+\int_0^1( \int_0^1 f(x,y)dy)^2 dx\leq ( \int_0^1\int_0^1 f(x,y)dxdy)^2+\int_0^1\int_0^1\,f(x,y)^2dxdy\) *

9 novembre 2022 22:31 — Par Patrice Lassère

Soit \(f\in\mathscr C^0([0,1]\times[0,1],\mathbb R)\), montrer que \[\begin{aligned}\int_0^1\left( \int_0^1 f(x,y)dx\right)^2 dy+&\int_0^1\left( \int_0^1 f(x,y)dy\right)^2 dx\\ &\leq \left( \int_0^1\int_0^1 f(x,y)dxdy\right)^2+\int_0^1\int_0^1\,f(x,y)^2dxdy.\end{aligned}\]



[ID: 2929] [Date de publication: 9 novembre 2022 22:31] [Catégorie(s): Suites et séries de fonctions, séries entières ] [ Nombre commentaires: 0] [nombre d'éditeurs: 1 ] [Editeur(s): Emmanuel Vieillard-Baron ] [nombre d'auteurs: 1 ] [Auteur(s): Patrice Lassère ]
Accordéon
Titre
Solution
Texte

\(\int_0^1( \int_0^1 f(x,y)dx)^2 dy+\int_0^1( \int_0^1 f(x,y)dy)^2 dx\leq ( \int_0^1\int_0^1 f(x,y)dxdy)^2+\int_0^1\int_0^1\,f(x,y)^2dxdy\)
Par Patrice Lassère le 9 novembre 2022 22:31

Considèrons pour \(m,n\in\mathbb Z\) \[\widehat{f}(n,m)=\int_0^1\int_0^1\,f(x,y)e^{-2i\pi(nx+my)}dxdy\] les coefficients de Fourier de \(f\). En particulier \[\widehat{f}(0,0)=\int_0^1\int_0^1 f(x,y)dxdy.{(1)}\] La fonction \(x\mapsto \int_0^1 f(x,y)dy\) est une fonction dont la série de Fourier est \(\sum_{n\in\mathbb Z}\widehat{f}(n,0)e^{-2i\pi nx}\). De carré intégrable sur \([0,1]\), on peut lui appliquer la formule de Parseval \[\int_0^1\left( \int_0^1 f(x,y)dy\right)^2 dx=\sum_{n\in\mathbb Z}\left\vert \widehat{f}(n,0)\right\vert^2.{(2)}\] De même \[\int_0^1\left( \int_0^1 f(x,y)dx\right)^2 dy=\sum_{m\in\mathbb Z}\left\vert \widehat{f}(0,m)\right\vert^2.{(3)}\] En appliquant Parseval dans \(L^2([0,1]^2)\) à \((x,y)\mapsto f(x,y)\) on a aussi \[\int_0^1\int_0^1\,f(x,y)^2dxdy=\sum_{n\in\mathbb Z}\sum_{m\in\mathbb Z}\left\vert \widehat{f}(n,m)\right\vert^2.{(4)}\] Avec \((1), (2), (3)\) et \((4)\) on a \[\begin{aligned}\left( \int_0^1\int_0^1 f(x,y)dxdy\right)^2+&\int_0^1\int_0^1\,f(x,y)^2dxdy-\int_0^1\left( \int_0^1 f(x,y)dx\right)^2 dy\\&-\int_0^1\left( \int_0^1 f(x,y)dy\right)^2 dx =\sum_{n\in\mathbb Z^\star}\sum_{m\in\mathbb Z^\star}\left\vert \widehat{f}(n,m)\right\vert^2\geq 0\end{aligned}\] Q.E.D.


Calcul de \(\int_0^\pi \cos(\cos(x))\rm{ch}(\sin(x))\cos(nx)dx, n\in\mathbb N\), via Fourier *

9 novembre 2022 22:31 — Par Patrice Lassère

Utiliser les séries de Fourier pour évaluer l’intégrale \[I_n=\int_0^\pi \cos(\cos(x)){\rm}{ch}(\sin(x))\cos(nx)dx,\quad n\in\mathbb N.\]



[ID: 2931] [Date de publication: 9 novembre 2022 22:31] [Catégorie(s): Suites et séries de fonctions, séries entières ] [ Nombre commentaires: 0] [nombre d'éditeurs: 1 ] [Editeur(s): Emmanuel Vieillard-Baron ] [nombre d'auteurs: 1 ] [Auteur(s): Patrice Lassère ]
Accordéon
Titre
Solution
Texte

Calcul de \(\int_0^\pi \cos(\cos(x))\rm{ch}(\sin(x))\cos(nx)dx, n\in\mathbb N\), via Fourier
Par Patrice Lassère le 9 novembre 2022 22:31

Tout calcul direct semble déraisonnable, on contourne cette difficulté en utilisant comme dans l’exercice ?????? les séries de Fourier. L’application régulière \(f(x)=\cos(\cos(x))\text{ch}(\sin(x))\) est \(2\pi\)-périodique paire, son coefficient de Fourier \(a_n\) vérifie donc \[a_n=\dfrac{2}{\pi}\int_0^\pi \cos(\cos(x))\text{ch}(\sin(x))\cos(nx)dx=\dfrac{2}{\pi}I_n.\] Il ne reste donc plus qu’à calculer \(a_n\). Pour cela écrivons \[\begin{aligned}f(x)&=\cos(\cos(x))\text{ch}(\sin(x))=\cos(\cos(x))\cos(i\sin(x))\\ &=\dfrac{1}{2}\left[ \cos(\cos(x)+i\sin(x))+\cos(\cos(x)-i\sin(x))\right] \\ &=\dfrac{1}{2}\left[ \cos(e^{ix})+\cos(e^{-ix})\right] =\sum_{k=0}^\infty \dfrac{e^{2ikx}+e^{-2ikx}}{2}\dfrac{(-1)^k}{(2k)!} =\sum_{k=0}^\infty\dfrac{(-1)^k}{(2k)!}\cos(2kx).\end{aligned}\] Cette dernière série étant normalement convergente sur \([0,2\pi]\), c’est la série de Fourier de \(f\), par conséquent \[I_n=\int_0^\pi \cos(\cos(x))\text{ch}(\sin(x))\cos(nx)dx=\begin{cases} \dfrac{\pi}{2}\dfrac{(-1)^k}{(2k)!}&\quad\text{si}\quad n=2k\\ 0&\quad\text{sinon.}\end{cases}\] Q.E.D.


Preuve du théorème des moments de Hausdorff par les séries de Fourier *

9 novembre 2022 22:31 — Par Patrice Lassère

Soit \(f\in\mathscr C^0([0,\pi])\) telle que \[\int_0^{2\pi}\,t^nf(t)dt=0,\quad\forall\,n\in\mathbb N.\] Montrer que \[\int_0^{2\pi}\,e^{int}f(t)dt=0,\quad\forall\,n\in\mathbb Z\] et en déduire que \(f\equiv 0\).



[ID: 2933] [Date de publication: 9 novembre 2022 22:31] [Catégorie(s): Suites et séries de fonctions, séries entières ] [ Nombre commentaires: 0] [nombre d'éditeurs: 1 ] [Editeur(s): Emmanuel Vieillard-Baron ] [nombre d'auteurs: 1 ] [Auteur(s): Patrice Lassère ]
Accordéon
Titre
Solution
Texte

Preuve du théorème des moments de Hausdorff par les séries de Fourier
Par Patrice Lassère le 9 novembre 2022 22:31

Notons \(\widetilde f\) la fonction \(2\pi\)-périodique paire continue sur \(\mathbb R\) qui coïncide avec \(f\) sur \(]0,2\pi]\) ; ses coefficients de fourier complexes sont donnés par \[c_n(\widetilde f)=\dfrac{1}{2\pi}\int_0^{2\pi}\,f(t)e^{int}dt,\quad \forall n\in\mathbb Z.\] Mais par normale convergence sur \([0,2\pi]\) de la série entière \(e^{int}=\sum_{k\geq 0}\frac{(int)^k}{k!}\) on peut écrire \[c_n(\widetilde f)=\dfrac{1}{\pi}\int_0^{2\pi}\,f(t)\left( \sum_{k\geq 0}\frac{(int)^k}{k!}\right)dt=\sum_{k\geq 0}\frac{(in)^k}{2\pi(2k)!}\int_0^{2\pi} t^{k}f(t)dt=0\] vu les hypothèses sur \(f\) : les coefficients de Fourier de l’application \(\widetilde f\) continue sur \([0,2\pi]\) et \(2\pi\)-périodique sont tous nuls, comme corollaire de la formule de Parseval1 elle est donc identiquement nulle et par suite \(f\). Modulo translation et homothétie le résultat en découle sur tout intervalle \([a,b]\) de \(\mathbb R\).


  1. 1  voir par exemple......

\(\sum_{k=0}^n C_{2n+2}^{2k+1}=2^{2n+1}(2n+1)(2n+2)\) via les séries entières *

9 novembre 2022 22:31 — Par Patrice Lassère

Calculer de deux manières différentes le développement en série entière à l’origine de \(f(x)=\sin^2(x)\) pour en déduire \[\sum_{k=0}^n C_{2n+2}^{2k+1}=2^{2n+1}(2n+1)(2n+2),\quad n\in \mathbb N^\star.{\text{($\star$)}}\]



[ID: 2935] [Date de publication: 9 novembre 2022 22:31] [Catégorie(s): Suites et séries de fonctions, séries entières ] [ Nombre commentaires: 0] [nombre d'éditeurs: 1 ] [Editeur(s): Emmanuel Vieillard-Baron ] [nombre d'auteurs: 1 ] [Auteur(s): Patrice Lassère ]
Accordéon
Titre
Solution
Texte

\(\sum_{k=0}^n C_{2n+2}^{2k+1}=2^{2n+1}(2n+1)(2n+2)\) via les séries entières
Par Patrice Lassère le 9 novembre 2022 22:31

\(f\) est bien entendu développable en série entière en \(x=0\) et on a pour \(x\in\mathbb R\) \[\begin{aligned} f(x)&=\sin^2(x)=\dfrac{1-\cos(2x)}{2}\\ &=\dfrac{1}{2}-\dfrac{1}{2}\sum_{n=0}^\infty (-1)^n\dfrac{(2x)^{2n}}{(2n)!}=-\sum_{n=1}^\infty (-1)^n\dfrac{2^{2n}x^{2n}}{2(2n)!}\\ &=\sum_{n=1}^\infty (-1)^{n+1} \dfrac{2^{2n-1}x^{2n}}{(2n)!} \end{aligned}\] soit \[\sin^2(x)=\sum_{n=1}^\infty (-1)^{n+1} \dfrac{2^{2n-1}}{(2n)!}x^{2n},\quad\forall\,x\in\mathbb R.{(1)}\] D’un autre coté, on peut écrire \[f(x)=\sin^2(x)=\left( \sum_{n=0}^\infty (-1)^n\dfrac{x^{2n+1}}{(2n+1)!}\right)^2=\sum_{n=1}^\infty c_{n} x^{2n}\] où par produit de Cauchy \[c_n=\sum_{k=0}^n \dfrac{(-1)^k}{(2k+1)!}\dfrac{(-1)^{n-k}}{(2n-2k+1)!} =\dfrac{(-1)^n}{(2n+2)!}\sum_{k=0}^n C_{2n+2}^{2k+1},\] donc \[\sin^2(x)=\sum_{n=1}^\infty \left( \dfrac{(-1)^n}{(2n+2)!}\sum_{k=0}^n C_{2n+2}^{2k+1}\right) x^{2n},\quad\forall\,x\in\mathbb R.{(2)}\] Par unicité des coefficients d’une série entière, (1) et (2) donnent \[\sum_{k=0}^n C_{2n+2}^{2k+1}=2^{2n+1}(2n+1)(2n+2),\quad n\in \mathbb N^\star,\] soit (\(\star\)), Q.E.D.

Remarque : Il n’est pas difficile d’établir directement (\(\star\)) par dénombrement car \(\sum_{k=0}^n C_{2n+2}^{2k+1}\) représente le nombre de parties de cardinal impair dans un ensemble de cardinal pair \(2n+2\).


;
Success message!